You are on page 1of 83

TYPE OF HOST

Definitive host Host in which the adult sexual phase of parasite development occurs
Intermediate host Host in which the larval asexual phase of parasite development occurs
Reservoir host Host harboring parasites that are parasitic for humans and from which humans may
become infected
Accidental or incidental Host other than the normal one that is harboring a parasite
host
Transport host Host responsible for transferring a parasite from one location to another
Carrier Parasite-harboring host that is not exhibiting any clinical symptoms but can infect others

TYPES OF PARASITE
Obligatory parasite Parasite that cannot survive outside of a host
Facultative parasite Parasite that is capable of existing independently of a host
Endoparasite Parasite that is established inside of a host
Ectoparasite Parasite that is established in or on the exterior surface of a host

PARASITE-HOST RELATIONSHIP TERMS


Symbiosis Living together; the association of two living organisms, each of a different species
Commensalism Association of two different species of organisms that is beneficial to one and neutral to the other
Mutualism Association of two different species of organisms that is beneficial to both
Parasitism Association of two different species of organisms that is beneficial to one at the other’s expense
Commensal Relating to commensalism; the association between two different organisms in which one benefits
and has a neutral effect on the other
Pathogenic Parasite that has demonstrated the ability to cause disease

PREVENTION AND CONTROL


Morbidity control Is the avoidance of illness caused by infections
Information-education- Is a health education strategy that aims to encourage people to adapt and maintain
communication healthy life practices
Environmental management Is the planning, organization, performance and monitoring of activities for the
modification and/or manipulation of environmental factors.
Environmental sanitation Involves interventions to reduce environmental health risk.
Sanitation Is the provision of access to adequate facilities for the disposal of human excreta;
usually combined with access to safe drinking water

SOURCES OF INFECTION
Contaminated soil Ascaris lumbricoides, Trichuris trichiura, Strongyloides stercoralis, and hookworms
Contaminated water Amoeba, Flagellates, cercariae of Schistosoma spp.
Arthropod vectors Malaria, Filarial parasites, Leishmania spp., Trypanosoma cruzi
Animals Toxoplasma gondii, Hymenolepis nana

MODE OF TRANSMISSION
Ingestion Foodborne: Taenia saginata, Taenia solium, Diphyllobothrium latum, Capillaria
philippinensis
Contaminated water: Entamoeba histolytica, Giardia lamblia
Skin penetration Hookworms, Strongyloides stercoralis, Schistosoma spp.
Vector bites Malaria, Filariasis, Leishmaniasis, Trypanosomiasis and Babesiosis
Congenital Transplacental: Toxoplasma gondii
transmission Transmammary: Ancyclostoma duoedenale, Strongyloides stercoralis
Inhalation Enterobius vermicularis
Sexual intercourse Trichomonas vaginalis

FIXATIVES
Formalin All-purpose fixative for the recovery of protozoa and helminths
Polyvinyl Alcohol Comprised of a plastic powder that acts as an adhesive for the stool specimen when
preparing slides for staining
Sodium Acetate Formalin Used for performing concentration techniques and permanent stained smears
(SAF)

FEATURES: P. vivax P. ovale P. malariae P. falciparum


Disease Benign tertian Benign tertian malaria Quartan malaria Black water fever,
associated malaria malignant tertian
malaria
Timing of cyclic Every 48 hours Every 48 hours Every 72 hours Every 36-48 hours
paroxysms
Relative age of Young RBCs Young RBCs Mature RBCs All ages of RBCs
infected RBCs
Mature schizont 12-24 merozoites Occupy 75% of RBCs 6-12 merozoites 8-36 merozoites
Rosette arrangement of arranged in rosette or (only detected in
merozoite (ave of 8 irregular clusters severe infections)
merozoite)
Morphologic Schuffner’s dot Schuffner’s dot (larger and Ziemann’s dot Maurer’s dot
features (eosinophilic darker than P. vivax)
(cytoplasm) stippling)

FILARIAL Common
Habitat: Disease: Specimen: Periodicity: Microfilaria: Vector:
WORMS: Name:
Sheated (no
Wuchereria Brancroft’s Lower Aedes, anopheles,
Elephantiasis Blood Nocturnal nuclei at tip of
bancrofti filaria lymphatics culex
tai)
Sheated (2
Brugia Malayan Upper Aedes, anopheles,
Elephantiasis Blood Nocturnal bulging nuclei
malayi filaria lymphatics mansonia
at tip)
Sheated
African Subcutaneous continuous Chrysops, Mango
Loa loa Loiasis Blood Diurnal
eyeworm tissue nuclei up to Fly, Tabanid Fly
the tip)
Unsheated (no
Onchocerca Blinding Subcutaneous River Non nuclei at the
Skin snips Simulium, Black Fly
volvulus filaria tissue blindness periodic tip, found only
in tissues)
Unsheated (no
Ozzard’s Subcutaneous
Mansonella Non Non nuclei at tip, Culicoides/Sucking
filaria/New tissue/ Body Blood
ozzardi pathogenic periodic found in midge, Simulium
World filaria cavities
blood)
Unsheated
Mansonella Perstan’s Non Non (nuclei up to
Body cavities Blood Culicoides
perstans filaria pathogenic periodic the tip of Blunt
tail
FEATURES: T. hominis T. tenax T. vaginalis
Habitat Intestine Oral cavity Vagina
Size Bigger Smallest Biggest
Nucleus 1 1 1
Undulating membrane Whole body 2/3 or ¾ ½ (shortest)
Inclusion bodies - - Siderophile granules
Specimen for diagnosis Stool Mouth scrapings Urine/Vaginal discharge
Mode of transmission Ingestion Oral transmission Sexual transmission
Motility Tumbling/Jerky Motility
SPECIMEN COLLECTION AND PROCESSING (21-22) Thick blood smears for malaria are recommended for
How many stool samples should be collected when species identification.
following the typical O&P collection protocol? A. True
A. 1 B. False
B. 2
C. 3 Giemsa is the preferred stain for the detection of blood
D. 4 parasites.
A. True
What is the purpose of fixatives for the collection of stool B. False
samples?
A. Enhance the motility of protozoa. Which of the following is the specimen of choice to
B. Stain the cytoplasmic inclusions of protozoa. demonstrate intracellular parasites such as Toxoplasma
C. Preserve the morphology of protozoa and prevent gondii and Leishmania spp.?
further development of helminths. A. Sputum
D. All of the above B. Urine
C. Tissue
Which of the following characteristics is observed during D. Genital secretions
the macroscopic examination of stool specimens?
A. Consistency The detection of an antibody to a given parasite in a patient
B. Color with no previous exposure prior to travel to an endemic area
C. Adult worms can be considered a positive result.
D. All of the above A. True
B. False
Which of these procedures is involved in the microscopic
examination of stool specimens for parasites? Which one of these parasites should be quantitated in the
A. Performing a concentration technique parasitology report?
B. Determining specimen consistency A. Giardia intestinalis
C. Examining sample for gross abnormalities B. Entamoeba coli
D. Analyzing sample for color C. Trichomonas vaginalis
D. Blastocystis hominis
The direct wet preparation can be eliminated from the O&P
examination if the specimen is received in a fixative. AMOEBAS (59-60)
A. True Amebas transform from trophozoites to cysts on entry into
B. False an unsuspecting human.
A. True
Which of the following parasitic stages is not usually B. False
detected after using a concentration technique?
A. Protozoan cysts Formed stool specimens are more likely to contain which of
B. Protozoan trophozoites the following?
C. Helminth eggs A. Trophozoites
D. Helminth larvae B. Cysts

The permanent stained smear is critical for detection of Infections with intestinal amebas are prevalent in which of
helminth eggs and larvae. the following?
A. True A. Underdeveloped countries with poor sanitary
B. False conditions
B. Beef consumers in the United States
What is one advantage of the stool screening method? C. People traveling to Europe
A. It is highly sensitive and specific. D. Japan, because of seafood diet
B. It can detect all parasites.
C. It can be performed on fresh or preserved specimens. Which of the following structures is (are) typical in
D. It is labor-intensive. trophozoites of E. histolytica?
A. Single nucleus with a small karyosome
From which area can the Enterotest be used to collect B. Unevenly distributed peripheral chromatin
specimens? C. Chromatoid bars
A. Duodenum D. Glycogen mass
B. Sigmoid colon
C. Stomach E. histolytica infection is traditionally diagnosed by
D. Perianal area finding which of the following? (Objective 3-8)
A. Adult and egg forms of the parasite in a suspected stool
sample
B. Trophozoites and/or cysts in a suspected stool Infection with E. polecki is mainly transmitted to humans via
sample which of the following?
C. Larvae in a suspected CSF sample A. Ingestion of E. polecki trophozoite
D. Adult form of the parasite in suspected tissue samples B. Ingestion of E. polecki cysts
C. Humans do not get infected with E. polecki.
The infective stage of E. histolytica is which of the D. Touching an injected pig or monkey
following?
A. Trophozoite Which of the following statements is not true about
B. Cyst the cysts of E. polecki?
A. Cysts vary in shape from spherical to oval.
Which of the following factors is not responsible for the B. Cysts contain at least four nuclei.
asymptomatic carrier state of a patient infected with E. C. The typical cyst nucleus resembles that of E. histolytica.
histolytica? D. A diffuse glycogen mass may be present in the
A. Low virulence strain cytoplasm of cysts.
B. Low inoculation into host
C. Intact patient’s immune system The appearance of an E. nana karyosome is usually which
D. Patient’s blood type of the following?
A. Granular
Which of the following prevention measures can control the B. Large and round
spread of E. histolytica? C. Small and round
A. Drinking tap water D. Blotlike
B. Using human feces as fertilizer
C. Boiling water or treat with iodine crystals Which of the following statements is true regarding E.
D. Practicing unsafe sex nana?
A. E. nana is found primarily in cold regions of the world.
Which of the following is true regarding E. hartmanni? B. E. nana is prevalent in areas in which poor hygiene
A. The organism is found worldwide. and substandard sanitary conditions exist.
B. It is generally considered a pathogen and treatment is C. E. nana is considered as a human pathogen, so
indicated. treatment is mandatory.
C. It is an extraintestinal ameba. D. Humans can obtain protection from E. nana via
D. The life cycle requires one morphologic form, the Vaccination.
trophozoite.
Iodamoeba bütschlii cysts typically:
A main difference between the trophozoites of E. hartmanni A. Contain four nuclei
and E. histolytica is which of the following? B. Have a small karyosome in a central position
A. Trophozoites of E. histolytica are smaller in size. C. Lack chromatoid bars
B. Presence of pseudopods D. Lack a glycogen mass
C. Trophozoites of E. hartmanni do not contain
ingested red blood cells. Transmission of I. bütschlii occurs when:
D. Nuclear structure and peripheral chromatin A. The infective cysts are ingested in contaminated
food or drink.
Which of the following is not true about E. coli? B. The infective trophozoites are ingested in contaminated
A. The parasite is found worldwide. food or drink.
B. It is considered to be a pathogen. C. Examining stool specimens from infected individuals
C. The infection is transmitted through the ingestion of the D. An infected mosquito vector bites an unsuspecting
infected cyst through contaminated food or drink. human.
D. The infection can be prevented by adequate disposal of
human feces and good personal hygiene practices. Which of the following is a unique characteristic of E.
gingivalis?
The trophozoites of E. coli: A. The trophozoites exhibit active motility via their
A. Have eight nuclei pseudopods.
B. Have a typical nucleus consists of a large, often B. There is no known cyst form of this parasite.
irregularly shaped karyosome that is eccentrically C. The trophozoite has a single nucleus with characteristics
located that resemble those of E. histolytica.
C. Are characterized by a karyosome that is surrounded by D. E. gingivalis is the only ameba that may ingest white
evenly distributed peripheral chromatin blood cells.
D. Contain red blood cell inclusions

E. gingivalis:
A. Is an intestinal ameba A. Aspiration or nasal inhalation of the organisms
B. Is a pathogen and must be treated with metronidazole C. B. Direct invasion of the parasites in the eye
Can be found in the mouth and in the genital tract C. Swimming or bathing in contaminated water
D. Has a typical amebic life cycle (i.e., trophozoites and D. A or B
cysts)
Infections with Acanthamoeba species are encountered in
The known morphologic forms of Naegleria fowleri are: which of the following anatomical parts? (Objective 3-5)
A. Ameboid trophozoites, flagellate forms, and cysts A. Eye
B. Ameboid trophozoites, immature cysts, and mature cysts B. Large intestines
C. Eggs, larvae, and adults C. Lungs
D. Flagellate forms, cysts, and larvae D. Liver

The specimen of choice for the recovery of N. fowleri is To prevent infection with Acanthamoeba species, contact
which of the following? lens wearers should avoid which of the following?
A. Sputum A. Strenuous exercise
B. Stool B. Foods with high carbohydrate content
C. Cerebrospinal fluid C. Wearing clothing made of cotton
D. Urine D. Using homemade nonsterile saline solutions

Humans most often contract N. fowleri by which of the FLAGELLATES (87-88)


following? All flagellate life cycles possess trophozoite and cyst
A. Swimming in contaminated water morphologic forms. (Objectives 4-5A and 4-5B)
B. Kissing an infected person A. True
C. Practicing unprotected sex B. False
D. Ingestion of contaminated food
This flagellate morphologic structure is often not visible
The ameboid trophozoites of N. fowleri enter the human under microscopic examination. (Objective 4-9A)
body through all of the following routes except: A. Undulating membrane
A. Entry through the nasal mucosa B. Pseudopods
B. Inhalation of contaminated dust C. Flagella
C. Sniffing contaminated water D. Axostyle
D. Ingesting contaminated food and drink
The presence of nonpathogenic flagellates is important
Practical measures for the control and prevention of N. because it suggests that:
fowleri include which of the following? A. The patient will develop clinical signs and symptoms.
A. Banning swimming at all times during the summer B. Only cyst forms will be recovered in corresponding
months patient samples.
B. Avoidance of chlorinating swimming pools and hot tubs C. The parasites will invade multiple organ systems in the
C. Providing education and awareness in the medical body.
community D. Contaminated food or drink was consumed by the
D. Avoidance of consuming contaminated food or water patient.
prior to swimming
The proposed function(s) of the median bodies seen in G.
The term acanthopodia refers to: intestinalis is (are) which of the following?
A. Spinelike pseudopods A. Support
B. Hairy projections B. Energy
C. Double-layer cell wall C. Metabolism
D. Large karyosome and no obvious peripheral chromatin D. All of the above

The specimen of choice for diagnosing Acanthamoeba Which specimen type and collection regimen would be
species trophozoites and cysts is which of the following? most appropriate for the diagnosis of G. intestinalis?
A. Urine A. One stool sample
B. Sputum B. Two stool samples
C. Cerebrospinal fluid C. Multiple stool samples collected on subsequent
D. Stool days
D. One stool sample and one blood sample

G. intestinalis trophozoites attach to the mucosa of the


duodenum and feed with the assistance of this morphologic
Humans may acquire Acanthamoeba species by which of structure.
the following? A. Sucking disk
B. Axostyle C. Ingestion of an embryonated ovum
C. Axoneme D. Ingestion of undercooked meat
D. Nucleus
When E. hominis cysts contain more than one nuclei, where
Individuals become infected with G. intestinalis by which of do they tend to be positioned within the cytoplasm?
the following? A. Center
A. Swimming in contaminated water B. Around the periphery of the organism
B. Ingesting contaminated food or drink C. At opposite ends of the cell
C. Inhalation of infective cysts D. Throughout the organism
D. Walking barefoot on contaminated soil
Treatment is always indicated for patients when E. hominis
Individuals at risk for contracting G. intestinalis when is present on parasite examination.
camping and hiking are encouraged to take which of these A. True
steps to prevent infection? B. False
A. Treat potentially infected water with a double strength
saturated saline solution prior to consuming. The traditional technique and specimen of choice for
B. Use only bottled water for drinking, cooking & identifying Retortamonas intestinalis is which of the
appropriate personal hygiene. following?
C. Avoid swimming in contaminated water. A. Permanently stained blood
D. Wear shoes at all times. B. Iodine prep of urine
C. Saline prep of bronchial wash
A liquid stool is the specimen of choice for the recovery of D. Permanently stained stool
which of these morphologic forms of C. mesnili?
A. Trophozoites only Individuals contract R. intestinalis by which of the following?
B. Cysts only A. Ingesting infective cysts in contaminated food or
C. Trophozoites and cysts drink
B. Consuming trophozoites in contaminated beverages
Which of the following are key morphologic characteristics C. Stepping barefoot on infective soil
of C. mesnili? D. Inhaling infective dust particles
A. Round and four to eight nuclei
B. Oval and presence of a cytosome How far down the body length does the Trichomonas tenax
C. Round and presence of an axoneme undulating membrane extend?
D. Lemon-shaped and presence of a cytosome A. One fourth
B. One half
A flagellate trophozoite that could be described as 9 to 12 C. Three fourths
µm with one or two nuclei, each with four symmetrically D. Full body
positioned chromatin granules and vacuoles containing
bacteria in the cytoplasm, would most likely be which of the The specimen of choice for the recovery of Trichomonas
following? tenax is which of the following?
A. Giardia intestinalis A. Stool
B. Dientamoeba fragilis B. Urine
C. Chilomastix mesnilli C. Mouth scrapings
D. Blastocystis hominis D. Cerebrospinal fluid

The permanent stain of choice for observing the nuclear This prominent structure found in T. vaginalis trophozoites
features of D. fragilis is which of the following? that often extends beyond the body provides the parasite
A. Trichrome with support.
B. Iodine A. Nucleus
C. Saline B. Axostyle
D. Iron hematoxylin C. Axoneme
D. Granule
The specimen of choice for the recovery of T. hominis is
which of the following? The cyst morphologic form is not known to exist in the life
A. Stool cycle of T. vaginalis.
B. Urine A. True
C. Intestinal contents B. False
D. Gastric contents T. vaginalis may be recovered in which of the following
Trichomonas hominis can be transmitted by which of the specimen types?
following? A. Spun urine
A. Contaminated milk B. Vaginal discharge
B. Bite of an infected mosquito C. Stool
D. Urethral discharge The vector responsible for the transmission of L. donovani
E. More than one of the above: A,B,C (specify) is:
A. Lutzomyia sandfly
All cases of T. vaginalis infection result in symptomatic B. Phlebotomus sandfly
vaginitis in women and urethritis in men. C. Psychodopygus sandfly
A. True D. None of the above
B. False
Which of the following items does not describe kalaazar?
Infant infections with T. vaginalis tend to affect which of the A. Commonly found in Iraq
following of these body areas? B. Transmitted by the Phlebotomus and Lutzomyia sandfly
A. Respiratory and genital species
B. Genital and intestinal C. Is not transmitted by blood transfusion
C. Intestinal and eye D. Can be serologically determined by ELISA, IFA, and
D. Respiratory and eye DAT methods

THE HEMOFLAGELLATES (113-114) The specimen of choice for the recovery of L. mexicana
This is the only hemoflagellate morphologic form that does complex members is:
not have an external flagellum. A. CSF
A. Trypomastigote B. Stool
B. Amastigote C. Lesion biopsy material
C. Promastigote D. Duodenal contents
D. Epimastigote
Which of the following is not a reservoir host for L. mexicana
Hemoflagellates are typically found in stool samples. complex?
A. True A. Squirrels
B. False B. Chipmunks
C. Rats
The symptoms of hemoflagellate infections range from D. Snakes
minor, such as irritation at the infection site, to serious
(comatose state and death). Which of the following best describe disease caused by the
A. True L. mexicana complex?
B. False A. Can disseminate into a diffuse cutaneous form
B. Appears around the ears in approximately 40% of
The organism causing chiclero ulcer is most likely found in: patients
A. Texas C. Both A and B are correct.
B. South American rainforest region D. None of the above
C. Nova Scotia
D. Egypt All the following are geographic regions in which the
members of the L. tropica complex can be found except:
Which of the following is not an acceptable treatment for A. Brazil
mucocutaneous leishmaniasis caused by L. braziliensis? B. Uzbekistan
A. Amoxicillin C. Iran
B. Pentosam D. Syria
C. Ambisome
D. Fungizone The specimen of choice for the recovery of L. tropica
complex members is:
Of the following laboratory diagnostic methods, which is the A. CSF
most commonly used for the recovery of members of the L. B. Fluid underneath the ulcer bed
braziliensis complex? C. Blood
A. Schizodeme analysis D. Tissue biopsy
B. Culture of infected material
C. Identifying amastigotes in infected material
D. Zymodeme analysis

The most common morphologic form seen in samples


A common name for disease caused by L. donovani is: positive for L. tropica complex members is:
A. Visceral leishmaniasis A. Trypomastigote
B. Kala-azar B. Promastigote
C. Dum dum fever C. Epimastigote
D. All of the above D. Amastigote
Of the following, which tests are considered diagnostic for The diagnostic testing methods for T. rangeli are the same
trypanosomiasis? as those for identifying and confirming and infection with T.
A. Giemsa-stained blood slides revealing the cruzi. (Objectives 5-8, 5-11)
trypomastigote A. True
B. Giemsa-stained blood slides revealing the amastigote C. B. False
Increased serum and CSF IgM levels
D. Both A and C are correct. The phrase that best describes the infection associated
E. All of the above with T. rangeli is that it: (Objective 5-6)
A. Mimics that of individuals infected with T. cruzi
There are no known animal reservoir hosts for T.b. B. Causes South American sleeping sickness
gambiense. C. Is considered a benign infection
A. True D. Produces Winterbottom’s sign
B. False
Which of the following is not a prevention and control
The enlargement of cervical lymph nodes in reference to measure for T. rangeli?
trypanosomal disease caused by T.b. gambiense is A. Use of DDT to control reduviid bug populations
referred to as: B. Better housing construction
A. Chancre C. Removing overgrown vegetation
B. Kerandel’s sign D. Educational programs in endemic areas
C. Winterbottom’s sign
D. Somnolence PLASMODIUM AND BABESIA (143)
The infective stage of Plasmodium is (are) the:
The diagnostic stage of T.b. rhodesiense is the: A. Merozoites
A. Trypomastigote B. Oocyst
B. Epimastigote C. Sporozoites
C. Promastigote D. Gametocytes
D. Amastigote
The best time to collect blood for Plasmodium parasites is:
Which of the following trypanosomal parasites that causes A. Between paroxysms
sleeping sicknesses is the more aggressive form? B. During paroxysms
A. Trypanosoma brucei gambiense C. Morning
B. Trypanosoma brucei rhodesiense D. Evening
C. Trypanosoma cruzi
D. Trypanosoma rangeli A paroxysm is:
A. An allergic reaction
What complicates the prevention and control efforts for T.b. B. A periodic episode characterized by fever, chills, sweats,
rhodesiense? and fatigue
A. Vast species of reservoir hosts C. Both A and B are correct.
B. Breeding occurs wherever brush is abundant D. None of the above
C. Both A and B
D. None of the above Which morphologic characteristic may help in
distinguishing P. vivax from P. falciparum?
The specimen of choice for the detection of T. cruzi is: A. Hemozoin
A. Stool B. Schüffner’s dots
B. Blood C. 72-hour paroxysm
C. Tissue D. None of the above
D. Ulcer
P. vivax characteristically invades:
Which of the following is the vector first identified as A. Immature RBCs
responsible for transmitting T. cruzi? B. Senescent RBCs
A. Phlebotomus spp. C. All RBCs
B. Lutzomyia spp. D. Lymphocytes
C. Panstrongylus megistus
D. Glossina spp.
Which of the following is not a characteristic finding in The incubation period for P. vivax is generally:
Chagas’ disease? A. 6 to 8 days
A. Romaña’s sign B. 7 to 10 days
B. Megacolon C. 12 to 24 days
C. Cardiomegaly D. 10 to 17 days
D. Somnolence
Which morphologic form would be the best choice for D. All of the above
distinguishing between P. vivax and P. ovale?
A. Mature schizont Humans are an accidental host of Babesia species.
B. Ring form A. True
C. Early trophozoite B. False
D. Immature schizont
The specimen of choice for the recovery of Babesia is:
In which geographic regions would the laboratorian most A. Tissue
likely suspect P. ovale as the infecting agent? B. Cerebral spinal fluid (CSF)
A. Tropical Africa C. Stool
B. Asia D. Blood
C. South America
D. All of the above Babesiosis is characterized by all the following except:
A. Trophozoites resembling the ring form seen in
Which of the following is considered an antimalarial Plasmodium infections
medication? B. A mild to severe hemolytic anemia
A. Amoxicillin C. Fever periodicity
B. Erythromycin D. None of the above
C. Chloroquine
D. Dicyclomine Which of the following are laboratory diagnostic procedures
is recommended for specifically identifying T. microti?
Which morphologic form is not typically seen in infections A. Thick and thin blood films
of P. malariae? B. Serologic testing
A. Mature schizont C. PCR techniques
B. Ring form D. Both B and C are correct.
C. Immature schizont E. None of the above
D. Macrogametocyte
Which of the following is not a location known for infection
Which of the following are morphologic features of P. by T. microti?
malariae? A. California
A. Schüffner’s dots B. North Carolina
B. Ziemann’s dots C. Mexico
C. Maurer’s dots D. Nantucket
D. None of the above
For which patient would babesiosis be more severe?
Which of the following is not a prevention and control A. The splenectomized
measure for malaria? B. The patient with Babesia divergens
A. Wearing the hair up C. Older adults
B. Following prophylactic therapy when traveling to malaria- D. All of the above
endemic areas
C. Bed netting MISCELLANEOUS PROTOZOA (172)
D. Proper clothing, such as long-sleeved shirt and long What makes each member of the the parasites dis-
pants cussed in this chapter similar?
A. Presence of cilia
What age of red blood cell does P. falciparum typically B. They are all unicellular.
invade? C. All are intestinal protozoa.
A. Mature red blood cells D. Presence of tachyzoites
B. Immature red blood cells
C. All red blood cells, regardless of age Which structure is always visible in the stained cyst
D. Does not invade red blood cells and troph of Balantidium coli? (Objective 7-9A)
A. Macronucleus
P. falciparum is commonly found in the United States. B. Micronucleus
A. True C. Cilia
B. False D. Ingested bacteria

Black water fever can be described by which of the The life cycle of Balantidium coli and clinical symp- toms
following: are similar to that of which of the following?
A. Marked hemoglobinuria A. Isospora belli
B. Kidney involvement in P. falciparum infections B. Entamoeba histolytica
C. Caused by P. falciparum–induced red blood cell C. Crytosporidium parvum
destruction D. Giardia intestinalis
A. Iron hematoxylin
Which two factors play an important role in the prognosis of B. Modifed acid-fast
a Balantidum coli infection? C. Gram
A. How infection occurred and duration of the infection D. Trichrome
B. Presence of coinfection and duration of the infection
C. Severity of infection and response to treatment All the following are recommended to prevent and
D. Immunocompetent status and severity of infection control an outbreak of Cryptosporidium except:
A. Proper treatment of water supplies
All the following are highly recommended when pro- B. Sterilize equipment using high heat.
cessing samples for the identification of Isospora belli to C. Sterilize equipment using full-strength bleach.
ensure identification except: D. Sterilize equipment using 5% to 10% household
A. Iodine wet prep ammonia.
B. Decreased microscope light level
C. Modified acid-fast stain Which is the best screening method for the identifica- tion
D. Saline wet prep of Blastocystis hominis?
A. Saline wet prep
Which stage of reproduction is considered capable of B. Modified acid-fast stain
initiating another infection of Isospora belli? C. Iodine wet prep
A. Sporozoites D. Iron hematoxylin stain
B. Immature oocysts
C. Merozoites Blastocystis hominis is always considered as being
D. Mature oocysts responsible for clinical symptoms when present in
human samples.
Which of the following patients would be more likely to A. True
contract an infection with Isospora belli? B. False
A. HIV-positive man
B. Female leukemia patient Which of the following measures that when taken can
C. Pig farmer prevent the spread of Blastocystis hominis?
D. Nursing home resident A. Avoid swimming in potentially contaminated water.
B. Proper sewage treatment
Which genus of parasite is most similar to Sarcocystis C. Use insect repellent.
based on morphologic similarities? D. Avoid unprotected sex.
A. Isospora
B. Blastocystis Diagnosis of Cyclospora can be accomplished by all
C. Entamoeba the following except:
D. Toxoplasma A. Concentration with formalin fixative
B. Flotation methods
How do humans become infected with Sarcocystis? C. Modified acid-fast stain
A. Ingestion of uncooked or undercooked beef or D. Addition of 5% potassium dichromate
pork
B. Inhalation of oocysts The clinical symptoms associated with Cyclospora
C. Ingestion of animal fecal contaminated food infections in children are similar to those seen in cases
D. More than one of the above: ________________ of infection by which of the following?
(specify) A. Naegleria
B. Cryptosporidium
In addition to oocysts, these Sarcocystis morphologic forms C. Leishmania
may be seen in human samples: D. Balantidium
A. Packets of eggs
B. Single or double sporocysts
C. Clusters of cysts
D. Groups of sporoblasts
The most important Cyclospora prevention step that
Which stage of reproduction is considered capable of can be taken is:
autoinfection of Cryptosporidium? A. Proper water treatment
A. Intact oocysts B. Wearing shoes when walking in sandy soil
B. Merozoites C. Insecticide treatment of mosquito breeding areas
C. Gametocytes D. Thoroughly cooking beef and pork.
D. Sporozoites
How do Microsporidia spores differ from other protozoan
The permanent stain of choice for the recovery of spores?
Cryptosporidium parvum is: A. Double outer wall
B. Extruding polar filaments B. Mosquito bite
C. Cilia C. Person-to-person
D. Pseudopods D. Hand-to-mouth

Of the following, which laboratory technique is required NEMATODES (203-204)


for species identification of Microsporidia? The junvenile stage of developing nematode worms is
A. Giemsa-stained biopsy material referred to as:
B. Electron microscopy A. Eggs
C. Fecal concentration B. Cysts
D. PAS stain C. Larvae
D. Adults
The life cycle of Microsporidia is a complex process in
which both the infective and diagnostic stages are Adult nematodes are never recovered in clinical samples.
spores. A. True
A. True B. False
B. False
These individuals are prone to experiencing severe
All the following are morphologic forms in the life cycle nematode infections:
of Toxoplasma gondii except: A. Completely healthy individuals with a heavy worm
A. Oocysts burden
B. Tachyzoites B. Unhealthy individuals who are asymptomatic
C. Bradyzoites C. Healthy individuals with a light worm burden
D. Sporozoites D. Unhealthy individuals with a heavy worm burden

Human infection of Toxoplasma is initiated in all the The specimen of choice for the recovery of Enterobius
following ways except: vermicularis is:
A. Accidental ingestion of rodent feces A. Stool
B. Ingestion of contaminated undercooked meat from B. Urine
cattle, pigs, or sheep C. Cellophane tape prep
C. Transplacental infection D. Tissue biopsy
D. Transfusion of contaminated blood
The most likely individual to contract pinworm infection
In which geographic area would you be likely to find is(are) a(an):
Toxoplasma gondii? A. Older adult
A. Tropics B. Child
B. Africa C. Adult prisoner
C. United States D. Military personnel
D. All of the above
The morphologic forms recovered in cases of pinworm
What is the preferred method of diagnosis for infection are which of the following?
Pneumocystis jiroveci? A. Eggs and adult females
A. Histologic stain B. Adult males and females
B. Giemsa stain C. Eggs and larvae
C. Iron hematoxylin stain D. Larvae and adult males
D. Iodine wet prep

Trichuris trichiura eggs are characterized by the presence


Which of the following groups of individuals is of which of the following?
considered at highest risk for contracting Pneumocystis A. Triple-layer cell wall
jiroveci? B. Flattened side
A. Veterans C. Hyaline polar plugs
B. Active military personnel D. Prominent cytostome
C. Immunosuppressed individuals
D. Newborns Of the following choices, the best laboratory diagno- sis
technique for the recovery of Trichuris trichiura eggs is
Pneumocystis jiroveci is believed to be spread via which which of the following?
of the following? A. Zinc sulfate flotation
A. Contaminated water B. Modified acid-fast stain
C. Ethyl acetate concentration The life cycle of Stronglyoides most resembles that of which
D. Wright-Giemsa stain of the following?
A. Pinworm
Children infected with Trichuris trichiura commonly suffer B. Whipworm
from: C. Hookworm
A. Mental confusion D. Threadworm
B. Hemoglobinuria E. Human roundworm
C. Severe anemia
D. Rectal prolapse The two clinical symptoms most commonly associated with
Trichinella spiralis are which of the following?
Individuals contract Ascaris lumbricoides via which of the A. Constipation and abdominal pain
following? B. Vomiting urticaria
A. Inhalation C. Diarrhea and vomiting
B. Insect bite D. Abdominal pain and diarrhea
C. Ingestion
D. Inappropriate sexual practices The specimen of choice for the identification of Trichinella
spiralis is which of the following?
The term that describes the lack of an outer mammil- lated A. Cerebral spinal fluid
albuminous coating is called which of the fol- lowing? B. Stool
A. Unfertilized C. Skeletal muscle
B. Fertilized D. Urine
C. Corticated
D. Decorticated The diagnostic stage of Trichinella spiralis is which of the
following?
The specimen of choice for the recovery of Ascaris A. Encysted larvae
lumbricoides eggs is which of the following? B. Cysts
A. Stool C. Eggs
B. Gallbladder biopsy D. Adult worms
C. Urine
D. Sputum Trichinosis is acquired via which of the following?
A. Swimming in contaminated water
This adult hookworm is characterized by a buccal cavity B. Consuming contaminated water
that contains teeth. C. Ingestion of contaminated food
A. Ancylostoma D. Inhalation of contaminated air droplets
B. Necator
C. Both Ancylostoma and Necator The two morphologic stages present in the Dracunculus
D. Neither Ancylostoma nor Necator medinensis are which of the following?
A. Eggs and larvae
Individuals contract hookworm via which of the fol- lowing? B. Larvae and adults
A. Contaminated water C. Eggs and adults
B. Skin penetration D. None of the above
C. Insect bite
D. Contaminated food

The specimen of choice for the recovery of Dracunculus


Which of the following are appropriate Ascaris lumbricoides medinensis is which of the following?
prevention and control strategies? A. Infected ulcer
A. Proper water treatment B. Skeletal muscle
B. Appropriate food handling C. Stool
C. Use of insect repellent D. Blood
D. Proper sanitation practices
When humans contract Dracunculus medinensis, the
This diagnostic stage of Strongyloides stercoralis is best parasite’s life cycle ceases to continue.
seen in stool using fecal concentration techniques: A. True
A. Eggs B. False
B. Rhabditiform larvae C. Unable to determine
C. Filariform larvae
D. Adult worms FILARIAE (223-224)
Speciation of the microfilariae can be done by recognition
of the distribution of nuclei in the tip of the tail and the What type of periodicity does Loa loa exhibit?
presence or absence of a delicate transparent covering A. Nocturnal
known as the: B. Subperiodic
A. Flilariform C. Diurnal
B. Cuticle D. None
C. Sheath
D. Nucleus How do the microfilariae of Onchocerca volvulus differ from
those of other filarial?
What type of periodicity is exhibited if microfilariae appear A. The presence of a sheath.
in the blood of an individual at 2:00 p.m. each day? B. Nuclei are present continuous to the end of the tail
A. Nocturnal C. They exhibit diurnal periodicity
B. Diurnal D. They are found in the skin rather than blood
C. Subperiodic
D. Biannual Skin snips are the specimen of choice for diagnosis of
infection with:
Which of the following is similar for all microfilariae A. Loa loa
discussed? B. Onchocerca volvulus
A. Presence of a sheath C. Brugia malayi
B. Ability to exhibit periodicity D. Wuchereria bancrofti
C. Location of the adult worms
D. The basic life cycle How are the microfilariae of Mansonella ozzardi
differentiated from those of Ochocerca volvulus?
Diagnosis of infection with Wuchereria bancrofti is best A. Location of the microfilariae
accomplished by: B. Absence of a sheath
A. Examination of stained peripheral blood taken C. Lack of terminal nuclei in the tail
during the night D. Presence of a sheath
B. Examination of stained tissue biopsy taken during the
night Which of the following filarial drugs is effective against
C. Use of serologic testing with blood taken during the day Mansonella ozzardi if therapy is necesary?
D. Examination of stained lymph fluid taken during the day A. Albendazole
B. Diethylcarbamazine
Which of the following, in combination with albendazole, C. Doxycycline
has proven to be an important drug for the treatment of D. Ivermectin
Bancroft’s filariasis?
A. Doxycycline If a physician suspects an individual has the potential for
B. Ivermectin infection with Mansonella perstans, what specimen type
C. Metronidazole would you suggest for recovery of the organism?
D. None of the above A. Skin snips
B. Lymphatic fluid
Which of the following can be used in the differentiation and C. Blood
identification of Brugia malayi? D. Infected nodules
A. Absence of a sheath
B. Absence of nuclei in the tail
C. Presence of a sheath that is very difficult to observe on
Giemsa stain
D. Presence of two terminal nuclei in the tail What type of periodicity does Mansonella perstans exhibit?
Select the ideal time period to collect blood samples for A. Nocturnal
examination for the presence of the microfilariae of Brugia B. Diurnal
malayi. C. Subperiodic
A. 10:00 p.m. to 4:00 a.m. D. None
B. 10:00 a.m. to 4:00 p.m.
C. 4:00 p.m. to 8:00 p.m. CESTODE (247)
D. Any time of the day or night The cestode morphologic form characterized by a
segmented appearance that houses male and female
A small threadlike worm is observed under the conjunctiva reproductive structures is referred to as a(an):
of the eye. What organism and form is most likely? A. Scolex
A. Wuchereria bancrofti adult B. Proglottid
B. Brugia malayi microfilaria C. Egg
C. Onchocera volvulus larva D. Cyst
D. Loa loa adult
Characteristics of the cestodes include all the follow- ing B. Effective rodent control
except: (Objective 10-9) C. Inspection of food prior to consumption
A. They are hermaphroditic. D. Protection of food from rodents
B. They generally require intermediate host(s).
C. Their laboratory diagnosis consists of finding larvae The characteristic of the life cycle of H. nana that
in feces. differentiates it from the other cestodes is which of the
D. Their anatomic regions include the scolex, neck, and following?
strobila. A. Lack of an intermediate host.
B. Infective larval stage.
A persistent cough, localized pain, and liver and lung C. Need for external environment
involvement are associated with an infection with which of D. Larval passage through the lungs
the following cestodes?
A. Diphyllobothrium latum A primary differential feature between an H. nana egg and
B. Echinococcus granulosus H. diminuta egg is which of the following?
C. Both A and B A. A flattened side for H. diminuta egg
D. Neither A nor B B. A thick shell for H. nana egg
C. Polar filaments in H. nana egg
Which of the following are key distinguishing factors in D. Radial striations in H. diminuta egg
differentiating an infection between T. saginata and T.
solium? Which of the following does not apply to H. nana?
A. Egg morphology and number of uterine branches in A. Dwarf tapeworm
proglottid B. Steatorrhea
B. Presence of hooklets on scolex and egg morphology C. Proper hygiene and sanitation procedures
C. Presence of hooklets and number of uterine D. Praziquantel therapy
branches in proglottid
D. Egg morphology and presence of suckers on scolex A unique characteristic of Dipylidium caninum is which of
the following?
The primary means of developing an intestinal infection A. Lack of suckers on the scolex
with Taenia spp. is via which of the following? B. Formation of egg packets
A. Skin penetration of larvae C. Proglottid resemblance to Taenia solium
B. Ingestion of raw or poorly cooked meat D. Alternation of female and male proglottids
C. Egg consumption
D. Drinking contaminated water A 2-year-old girl and her pet dog were diagnosed with D.
caninum infection. This infection was acquired by which of
Which is the preferred drug for treating intestinal infection the following?
by Taenia spp.? A. Ingestion of the parasite’s egg
A. Praziquantel B. Penetration of soil larva
B. Penicillin C. Ingestion of a flea
C. Nicolasamide D. Consumption of poorly cooked beef
D. Pentamidine

Prevention and control measures to prevent D. caninum


Which of the following is characteristic of an H. diminuta infection include all except which of the following?
egg? A. Treat dog and cat pets to prevent fleas.
A. Spherical, with radial striations B. Warn children against dog and cat licks.
B. Ellipsoid, with terminal polar plugs C. Deworm dog and cat pets, as needed.
C. Oval, with thin shell and polar filaments D. Neuter dog and cat pets.
D. Oval, with polar thickenings and no filaments
The egg of D. latum is unique among the cestodes in that it
The infective stage of H. diminuta for humans is which of contains which of the following?
the following? A. An operculum and terminal knob
A. Rhabditiform larva B. Radial striations and oncosphere
B. Cysticeroid larva C. An operculum and lateral spine
C. Embryonated D. A ciliated rhabditiform larva
D. Encysted form
Which of the following associations is correct for D. latum?
Prevention and control measures against H. diminuta A. Snail-coracidium
include all except which of the following? B. Copepod-procercoid
A. Vaccination program C. Fish-cysticercus
D. Beetle-pleurocercoid The eggs of Fasciolopsis and Fasciola possess a caplike
structure from which their contents are released under the
The primary pathology associated with a D. latum infection appropriate conditions; this is called a (an):
is which of the following? A. Operculum
A. Eosinophilic pneumonitis B. Shoulder
B. Vitamin D deficiency C. Coracidium
C. Vitamin B12 deficiency D. Digenea
D. Fat malabsorption
The number of Clonorchis cases has tripled in which
Which of the following procedures would not be appropriate country because of aquaculture?
for diagnosing an infection with Echinococcus granulosus? A. Japan
A. Serologic procedure, such as ELISA B. Vietnam
B. O&P examination of stool specimen C. China
C. CT scan of suspect organ D. Korea
D. Biopsy of cyst
What procedures must be done to recover the adult form of
In humans, Echinococcus granulosus infection results in Clonorchis sinensis?
which of the following? A. Direct examination of stool and after autopsy
A. Eggs similar to those of H. nana B. Direct and concentration examinations of stool
B. A nutritional deficiency C. Following surgery and after autopsy
C. A hydatid cyst D. Following surgery and duodenal aspiration
D. Filariform larva
Which of the following is a recommended prevention and
Which of the following is not a usual site for Echinococcus control strategy designed to halt the spread of Clonorchis?
granulosus infection in humans? A. Consuming raw, pickled, freshwater fish
A. Brain B. Protecting food from flies
B. Liver C. Avoidance of swimming in fresh water
C. Lung D. Proper human and reservoir host fecal disposal
D. Genitalia
The key feature that distinguishes Heterophyes from
TREMATODES (269-270) Clonorchis is which of the following?
The first intermediate host for all the trematodes is which of A. Size
the following? B. Shape
A. Fish C. Appearance of shoulders
B. Snail D. Location of operculum
C. Shrimp
D. Water plant The treatment of choice for Heterophyes and Meta-
gonimus is which of the following? (Objective 11-7A)
Adult trematodes are readily recoverable in clinical A. Niclosamide
samples. B. Praziquantel
A. True C. Pyrantel pamoate
B. False D. Metronidazole
Individuals suffering from trematode infections experience The specimen of choice for the recovery of Heterophyes
a variety of species-dependent symptoms. and Metagonimus is which of the following?
A. True A. Stool
B. False B. Duodenal contents
C. Urine
Fasciolopsis buski infects which organ in humans? D. Sputum
A. Bile ducts
B. Liver In addition to its typical location, Paragonimus eggs are
C. Colon also known to cause serious complications when recovered
D. Small intestine in which of the following?
A. Bile
The determination of Fasciolopsis versus Fasciola can only B. Cerebrospinal fluid
be accomplished in the laboratory by the recovery of which C. Brain tissue
of the following? D. Feces
A. Eggs
B. Larvae A key feature that distinguishes Paragonimus from the
C. Adults other trematode eggs is which of the following?
D. Sporocysts A. Prominent operculum
B. Obvious terminal shell thickening
C. Discrete shoulders Association of two different species or organisms that is
D. Three pairs of hooklets beneficial to both.
A. Commensalism
The typical transmission route of Paragonimus to humans B. Mutualism
consists of which of the following? C. Parasitism
A. Consumption of contaminated crayfish or crabs D. Pathogenic
B. Swimming in contaminated water
C. Hand-to-mouth contamination The most common source of parasitic infection.
D. Walking barefoot on contaminated sandy soil A. Arthropod vectors
B. Contaminated food
The adults of this species of Schistosoma dwell in the veins C. Contaminated soil and water
surrounding the urinary bladder: D. Fomites
A. S. haematobium
B. S. mansoni All of the following are correctly matched according to their
C. S. japonicum mode of transmission, EXCEPT:
D. All of the above a. Foodborne - Diphyllobothrium latum
b. Skin penetration - Strongyloides stercoralis
The specimen of choice for the recovery of Schistosoma c. Vector bites - Wuchereria bancrofti
japonicum is which of the following? d. Contaminated water - Toxoplasma gondii
A. Tissue biopsy
B. Urine A morphologic form that can be detected via laboratory
C. Sputum retrieval methods.
D. Stool A. Infective stage
B. Diagnostic stage
A systemic hypersensitivity reaction caused by the C. Both of these
presence of Schistosoma is called which of the following? D. None of these
A. Bilharziasis
B. Katayama fever It involves interventions to reduce environmental health
C. Swamp fever risks.
D. Schistosomiasis A. Environmental management
B. Morbidity control
C. Environmental sanitation
PARA RATIONALE D. Information-education-communication
Type of parasite that cannot survive outside of a host.
A. Obligatory parasite How many stool samples should be collected when
B. Facultative parasite following the typical ova and parasite (O&P) collection
C. Endoparasite protocol?
D. Ectoparasite A. 1
B. 2
Host in which the adult sexual phase of parasite C. 3
development occurs. D. 4
A. Accidental host
B. Definitive host When using preservatives, what is the appropriate ratio of
C. Carrier fixative to stool?
D. Intermediate host A. One part fixative to one part stool
B. Two parts fixative to one part stool
Parasite-harboring host that is not exhibiting any clinical C. Three parts fixative to one part stool
symptoms but can infect others. D. Four parts fixative to one part stool
A. Accidental host
B. Definitive host All-purpose fixative for the recovery of protozoa and
C. Carrier helminths.
D. Intermediate host A. Formalin
B. Saline
C. Polyvinyl Alcohol
Association of two different species of organisms that is D. Sodium acetate formalin
beneficial to one and neutral to the other.
A. Commensalism The preferred specific gravity of zinc sulfate solution for the
B. Mutualism flotation method is:
C. Parasitism A. 1.01
D. Pathogenic B. 1.04
C. 1.18
D. 1.48 The specimen of choice for the recovery of Trichomonas
hominis is which of the following?
In the collection and transport of stool specimen for A. Stool
parasites, which parasitic stage is most affected by the B. Mouth scrapings
length of time from collection to examination? C. Urine
A. Cysts D. Cerebrospinal fluid
B. Trophozoites
C. Oocyts Contains single nucleus and fused fibrils forming a
D. Helminth larvae characteristic of “bird’s beak” appearance.
a. Retortamonas intestinalis
The formalin-ethyl acetate concentration method for feces b. Enteromonas hominis
is used to demonstrate: c. Dientamoeba fragilis
A. Formation of amoebic pseudopodia d. Chilomastix mesnii
B. Motility of helminth larvae
C. Hatching larval forms The specimen of choice for the recovery of Trichomonas
D. Protozoan cysts and helminth eggs tenax is which of the following?
A. Stool
Designated as "small race" of Entamoeba histolytica. B. Mouth scrapings
A. Entamoeba gingivalis C. Urine
B. Entamoeba hartmanni D. Cerebrospinal fluid
C. Entamoeba polecki
D. Endolimax nana Relative age of RBCs that are infected by Plasmodium
vivax.
Only pathogenic intestinal amoeba. a. young/immature
A. Entamoeba gingivalis b. Mature cells
B. Naegleria fowleri c. Both a and b
C. Entamoeba histolytica d. None of the above
D. Acanthamoeba spp.
The red blood cells infected with Plasmodium falciparum
Only amoeba that ingests white blood cells. contains dark-staining, irregular to comma-shaped
A. Entamoeba gingivalis cytoplasmic dots called ____.
B. Entamoeba coli a. Schuffner’s dot
C. Entamoeba histolytica b. Zieman’s dot
D. Iodamoeba butschlii c. Maurer’s dot
d. All of the above
Primary amoebic meningoencephalitis (PAM) is primarily
caused by: Disease associated to Plasmodium malariae:
A. Entamoeba histolytica a. Benign tertian malaria
B. Iodamoeba butschlii b. Black water fever
C. Naegleria fowleri c. Malignant tertian malaria
D. Acanthamoeba spp. d. Quartan malaria

Agent of Granulomatous Amoebic Encephalitis (GAE): Black water fever can be described by which of the
A. Entamoeba histolytica following:
B. Iodamoeba butschlii a. Marked hemoglobinuria
C. Naegleria fowleri b. Kidney involvement
D. Acanthamoeba spp. c. P. falciparum-induced red blood cell destruction
d. All of the above
It is initially known as Cercomonas intestinalis
A. Enteromonas hominis Once thought to be solely a parasite of “Old World
B. Giardia lamblia Monkeys”
C. Retortamonas intestinalis a. Plasmodium vivax
D. Dientamoeba fragilis b. Plasmodium malariae
c. Plasmodium falciparum
Lemon-shaped cyst with a clear hyaline knob extending d. Plasmodium knowlesi
from the anterior end.
A. Chilomastix mesnii The vector most commonly associated with the
B. Dientamoeba fragilis transmission of Babesia microti
C. Retortamonas intestinalis a. Ixodes dammini
D. Trichomonas hominis b. Ixodes ricinus
c. Both of these
d. None of these
It is also known as the “Old World hookworm”
The most common tapeworm recovered in the Unites a. Ascaris lumbricoides
States. b. Dracunculus medinensis
A. Taenia saginata c. Ancylostoma duodenale
B. Taenia solium d. Necator americanus
C. Hymenolepis diminuta
D. Hymenolepis nana The specimen of choice for the recovery of Enterobius
vermicularis.
Risk for developing vitamin B12 deficiency is associated a. Stool
with patients infected with ____. b. Urine
a. Dipylidium caninum c. Cellophane tape prep
b. Taenia saginata d. Tissue biopsy
c. Diphyllobothium letum
d. Hymenolepis nana Children infected with Trichuris trichiuria commonly suffer
from:
In the Diphyllobothrium latum life cycle, the infective stage a. Mental confusion
for humans is: b. Hemoglobinuria
a. Cysticercus c. Severe anemia
b. Cysticercoid d. Rectal prolapse
c. Procercoid
d. Plerocercoid The mouth of Necator americanus is characterized by the
presence of
Cysticercosis is caused by the disseminated larva of: a. Teeth
a. Hymenolepis nana b. Cutting plates
b. Taenia solium c. Both
c. Hymenolepis diminuta d. Neither
d. Diphyllobothrium latum
The adult hookworm is characterized by a buccal cavity that
Stool specimen contains tapeworm segment, uterus of contains teeth:
which has more than 15 lateral branches. a. Ancylostoma duodenale
a. Taenia solium b. Necator americanus
b. Dipylidium caninum c. Both
c. Taenia saginata d. Neither
d. Hymenolepis nana
Which of the following microfilariae is sheathed with
Packets of tapeworm eggs encapsulated in a single continuous nuclei up to the tip?
membrane were recovered in feces. a. Wuchereria brancrofti
a. Dipylidium caninum b. Brugia malayi
b. Taenia saginaa c. Loa loa
c. Hymenolepis nana d. Onchocerca volvulus
d. Taenia solium
Larval stage of Echinococcus granulosus Which of the following filarial worms causes River blindness
a. Cysticercus disease?
b. Cysticercoid a. Wuchereria brancrofti
c. Hyatid cyst b. Brugia malayi
d. Plerocercoid c. Loa loa
d. Onchocerca volvulus
It is known as the great imitator because infected patients
may experience a variety of symptoms that often mimic Skin snips are the specimen of choice for diagnosis of
those other diseases and conditions. infection with:
a. Trichinella spiralis a. Loa loa
b. Strongyloides stercoralis b. Brugis malayi
c. Ascaris lumbricoides c. Mensonella ozzardi
d. Enterobius vermicularis d. Onchocerca volvulus

Considered as one of the largest adult nematodes. The fluke acquired by eating contaminated vegetation is:
a. Ascaris lumbricoides a. Clonorchis sinensis
b. Dracunculus medinensis b. Fasciolopsis buski
c. Ancylostoma duodenale c. Paragonimus westermani
d. Necator americanus d. Schistosoma mansoni
c. Speaker of the house
The typical transmission route of Paragonimus to humans? d. Secretary of health
a. Consumption of contaminated crayfish or
crabs A laboratory performing culture and sensitivity is a:
b. Hand to mouth contamination a. Primary laboratory
c. Swimming in contaminated water b. Secondary laboratory
d. Walking barefoot on contaminated sandy soil c. Tertiary laboratory
d. Special laboratory
MTLBE RATIONALE
Which of the following can head a primary or secondary
Father of PAMET laboratory in area wherein the number of pathologists is
a. Felix Asprer insufficient?
b. Crisanto Almario A. Medical technologist
c. Charlemagne Tamondong B. Chemist
d. Gustavo Reyes C. Physician even without a training
D. Physician with at least 3 months training in
RA 5527 was approved on: laboratory medicine
a. June 21, 1969
b. July 21, 1969 The records of forensic pathology shall be kept in the
c. June 21, 1968 laboratory for:
d. June 11, 1978 A. 5 years
B. 10 years
The minimum required course of the Philippine Medical C. 15 years
Technology Act of 1969 is stated in Section: D. Permanently
a. 5
b. 6 Philippine AIDS Prevention and Control Act of 1998:
c. 10 A. RA 7719
d. 15 B. RA 8504
C. RA 1517
The composition of the Medical technology board: D. PD 223
a. Pathologist, medical technologist, medical
technician The chairman and members of the Board of Medical
b. Secretary of health, dean, president of PAMET Technology shall hold office for __ years after appointments
c. Pathologist, medical technologist, intermist or until successors shall have been appointed and duly
d. Registered pathologist, two registered medical qualified.
technologist A. 1 year
B. 2 years
Administrative investigation shall be conducted by: C. 3 years
a. At least one member of the board D. 10 years
b. Three members of the board
c. At least one member of the board with one legal Rating in the Examination in order to pass:
officer 1. Obtained a general average of at least 75%
d. At least two members of the board with one 2. No rating below 50% in any of the major subjects
legal officer 3. Not failed in at least 60% of the subjects computed
according to their relative weights
The penalty of revocation of certificate of registration may
be imposed by the board if there is: A. Only 1
a. Majority vote B. 1 and 2
b. Unanimous vote C. 1 and 3
c. Either of these D. 1, 2 and 3
d. Neither of these
All successful examinees will be required to take their
A roster of MT is prepared: professional oath:
a. Annually by the PRC commissioner A. Before the president of the philippines
b. Annually by board members B. Before a judge
c. Annually by the board discuss C. Before the PAMET President
d. Annually by the secretary of the board D. Before the Board of MT or any person
authorized to administer oath
The chairperson of PRC is appointed by:
a. President of the philippines Administrative investigation shall be conducted by:
b. Sentare president A. At least one member of the Board
B. Three members of the Board accreditation from ___ which shall set and maintain
C. At least one member of the Board with one legal reasonable accreditation standards.
officer A. PAMET
D. At least two members of the Board with one B. DOH
legal officer C. DECS
D. TESDA
No penalty of revocation shall be imposed on a medical
technologist, unless: National Reference Laboratory for dengue, influenza,
A. There is a vote from the chair and one member of malaria; NRL for confirmatory testing of blood units
the Board of medical technology A. RITM
B. There is a majority vote from the board of medical B. EAMC
technology C. NKTI
C. There is a unanimous vote from the Board of D. SLH
medical technology
D. There is at least one vote from any members of the It refers to procedures to account for each specimen by
Board of medical technology tracking its handling and storage from point of collection to
final disposal.
The clinical laboratory law requires that: A. Manual
A. The pathologist owns the clinical laboratory B. Confirmatory test
B. The clinical laboratory is owned by a medical C. Chain of infection
technologist D. Chain of custody
C. The clinical laboratory has a pathologist, medical
technologist and medical laboratory technician All of the following statements are true regarding the
D. The clinical laboratory is headed by a types of parasites, EXCEPT:
pathologist A. Facultative parasite is capable of existing
independently of a host
An individual who can be a head of a clinical laboratory: B. Obligatory parasite can survive outside the
A. Medical technologist host
B. Pathologist C. Ectoparasite is established in or on the exterior
C. Clinician surface of a host
D. Registered MT D. Endoparasite is established inside of a host

The blood banking laws of 1956: All of the following statements are true regarding the
A. RA 5527 types of hosts, EXCEPT:
B. RA 4688 A. Intermediate host is where the larval asexual
C. RA 1517 phase of parasite development occurs
D. RA 7719 B. Transport host is responsible for transfering a
parasite from one location to another
According to AO 2008-0008, services done in a blood C. Accidental host is a host other than the normal
station, except: one that is harboring a parasite
A. Provision of whole blood and PRBC’s D. Reservoir host is a parasite-harboring host that
B. Storage, transport and distribution of WB and is not exhibiting any clinical symptoms but can
PRBC’s infect others
C. Compatibility testing of red cell units, if hospital-
based Host in which the adult sexual phase or parasite
D. Screening and selection of voluntary blood development occurs
donors A. Accidental host
B. Definitive host
According to the provisions of RA 8504, all teachers, and C. Carrier
instructors of HIV/AIDS courses shall be required to D. Intermediate host
undergo a seminar or training on HIV/AIDS prevention and
control to be supervised by ____, in coordination with the Which of the following statements are true regarding
Department of Health (DOH) before they are allowed to parasite-host relationship:
teach on the subject. 1. Symbiosis is the association of two living
A. DECS organisms, each of a different species
B. TESDA 2. The primary function of the host is to carry on the
C. CHED parasite's life cycle
D. Any of these 3. Pathogenic parasite has demonstrated the ability to
cause disease
All testing centers, hospitals, clinics and laboratories 4. Mutualism is the association of two different
offering HIV testing services are mandated to seek species of organisms that is beneficial to both
A. 1, 3 and 4 B. Polyvinyl Alcohol
B. All of the above C. Saline
C. 2, 3 and 4 D. Sodium Acetate Formalin
D. None of the above
Zinc sulfate flotation technique:
Association of two different species of organisms that ● Zinc sulfate with a specific gravity of 1.48 to 1.50 is
is beneficial to one and neutral to the other. used as the concentration solution
A. Commensalism ● The parasites float to the surface and can be
B. Parasitism SkImmed from (ne (op or the tube
C. Mutualism ● It yields a cleaner preparation
D. Pathogenic ● Some helminth eggs are very dense and will not
float
The most common source of parasitic infection: A. 2 statements are correct
A. Arthropod vectors B. 3 statements are correct
B. Contaminated Food C. All statements are correct
C. Contaminated soil and water D. None of the statements correct
D. Fomites
In the collection and transport of stool specimens for
All of the following are correctly matched according to parasites, which parasitic stage is most affected by the
their mode of transmission, EXCEPT: length of time from collection to examination?
A. Foodborne - Diphyllobothrium latum A. Cysts
B. Skin penetration - Strongyloides stercoralis B. Oocysts
C. Vector bites - Wucherena bancrofti C. Trophozoites
D. Contaminated water - Toxoplasma gondii D. Helminth larvae

Infective stage is a morphologic form that can be Formalin-Ethyl Acetate Sedimentation Procedure:
detected via laboratory retrieval methods. Diagnostic 1. This is the most widely used sedimentation
stage is described as the form that can be detected via technique
laboratory retrieval methods. 2. The principle of this technique is based on specific
A. First statement is Correct gravity
B. Second statement is correct 3. More fecal debris is removed
C. Both statements are correct 4. Provides good recovery of most parasite and easy
D. Both statements are incorrect to perform
A. 1 and 2
Involves interventions to reduce environmental health B. 2 and 4
risks. . C. 2,3 and 4
A. Environmental management D. 1,2 and 4
B. Morbidity control
C. Environmental sanitation Which of the following statements regarding Entamoeba
D. Information-education-communication hartmanni is true?
How manu stool samples should be collected when A. Designated as small race of Entamoeba
following the typical ova and parasite (O&P) collection histolytica
protocol? B. Survival in a feces-contaminated environment for
A. 1 up to one month
B. 2 C. Both A and B
C. 3
D. 4 Only pathogenic intestinal amoeba.
A. Entamoeba gingivalis
The ratio of fixative to stool is important for the B. Entamoeba histolytica
successful recovery of parasites. Whatever fixative is C. Naegleria fowleri
used, the recommended ratio is five parts fixative to D. Acanthamoeba spp.
one part Stool
A. First statement is correct Only amoeba that ingests white blood cells.
B. Second statement is correct A. Entamoeba gingivalis
C. Both statements are correct B. Entamoeba histolytica
D. Both statements are incorrect C. Entamoeba coli
D. lodamoeba butschli
This preservative can be used for performing
concentration techniques and permanent stained All of the following statement is true regarding Naegleria
smears fowleri, EXCEPT?
A. Formalin
A. The only ameba with three known morphologic B. Rosette arrangement of merozoites
forms C. Paroxysms typically occur every 48 hours
B. Causes Granulomatous amebic encephalitis D. Contain tiny granules in the cytoplasm known as
C. Microscopic examination of CSF is the method of Schuffner's dots
choice for the recovery of N. fowleri ameboid
trophozoites The red blood cells infected with Plasmodium falciparum
D. The cyst form is known to exist only in the external contains dark-staining, irregular to comma-shaped
environment cytoplasmic dots called
A. Schuffner's dot
Agent of Granulomatous Amoebic Encephalitis (GAE): B. Maurer's dot
A. Entamoeba histolytica C. Zieman's dot
B. Naegleria fowleri D. All of the above
C. lodamoeba butschlit
D. Acanthamoeba spp. Plasmodium malaria primarily infects mature red blood
cells. Mature schizont of Plasmodium malaria typically
All of the following statements are true regarding Giardia contains 6 to 12 merozoites arranged in rosettes.
Intestinalis, EXCEPT: A. First statement is correct
A. is initially known as Cercomonas intestinalis B. Second statement is correct
B. is considered to be one of the most common C. Both statements are correct
intestinal parasites D. Both statements are incorrect
C. It is initially classified as an ameba
D. It is considered to be the only known pathogenic Black water fever can be described by which of the
intestinal flagellate following:
A. Marked hemoglobinuria
Axostyle is a finlike structure connected to the outer edge B. Kidney involvement
of some flagellates. Undulating membrane is a rodlike C. P. falciparum-induced red blood cell destruction
support structure found in some flagellates. D. All of the above
A. First statement is correct
B. Second statement is correct Once thought to be solely a parasite of "Old World
C. Both statements are correct Monkeys"
D. Both statements are incorrect A. Plasmodium vivax
B. Plasmodium falciparum
The specimen of choice for the recovery of Trichomonas C. Plasmodium malaria
hominis is which of the following? D. Plasmodium knowlesi
A. Stool
B. Urine The vector most commonly associated with the
C. Mouth scrapings transmission of Babesia microti
D. Cerebrospinal fluid A. Ixodes dammini
B. Both of these
Contains single nucleus and fused fibrils forming a C. Ixodes Ricinus
characteristic of "bird's beak appearance. D. None of these
A. Retortamonas intestinalis
B. Enteromonas hominis All of the following statements are true regarding
C. Dientamoeba fragilis Hymenolepis nana, EXCEPT:
D. Chilomastix mesnili A. Also known as the rat tapeworm
B. Praziquantel is considered to be the treatment of
Trichomonas tenax: choice
● Specimen of choice is mouth scrapings C. Considered to be the most common tapeworm
● Undulating membrane extending only half of the recovered in the United States
● body length D. Scolex contains four suckers and a short rostellum
● Smallest among the Trichomonas spp. with one row of hooks
● There is no known cyst stage
A. 1 statement is correct Risk for developing vitamin B12 deficiency is associated
B. 2 statements are correct with patients infected with
C. 3 statements are correct A. Dipylidium caninum
D. All statements are correct B. Diphyllobothrium latum
C. Taenia saginata
All of the following statements are true regarding D. Hymenolepis nana
Plasmodium vivax, EXCEPT:
A. Patients infected with P. vivax typically begin to
develop the symptoms of Benign Tertian Malaria
1

CHAPTER 3: AMOEBA TROPHOZOITES


Parameters E. histolytica E. hartmanni E. coli E. polecki E. nana I. bütschlii E. gingivalis N. fowleri Acanthamoeba
Size 8-65 μm 5-15 μm 12-55 μm 8-25 μm 5-12 μm 8-22 μm 8-20 μm 8-22 μm 12-45 μm
Motility Progressive, Nonprogressive, Nonprogressive, Sluggish, Sluggish, Sluggish, usually Active, varying Sluglike, blunt Sluggish,
finger-like finger-like blunt nonprogressive nonprogressive, progressive pseudopod pseudopods spinelike
pseudopodia pseudopods pseudopods blunt pseudopods appearance pseudopods
Motility - - - Progressive, - - - - -
diarrheal unidirectional
stools
No. of nuclei 1 1 1 1 1 1 1 1 1

Karyosome Small and central Small and central Large, irregular Small and central Large, irregular, Large, usually Centrally located Large and usually Large
shape, blotlike central refractive centrally
eccentric achromatic located
granules may or
may not be
present

Peripheral Fine and evenly Fine and evenly Unevenly Fine and evenly Absent Absent Fine and evenly Absent Absent
Chromatin distributed distributed distributed distributed distributed
Cytoplasm Finely granular Finely granular Coarse and Granular and Granular and Coarsely granular Finely granular Granular, usually Granular and
granulated vacuolated vacuolated and vacuolated vacuolated
vacuolated
Cytoplasmic Ingested RBCs Ingested bacteria Vacuoles Ingested bacteria Bacteria Bacteria Leukocytes
inclusions may be containing bacteria Other food Yeast cells Epithelial cells
present often visible particles Other debris Bacteria

Pictures

PARASITOLOGY RMT SOON! <3 ADM


2

CHAPTER 3

AMOEBA CYST
Parameters E. histolytica E. hartmanni E. coli E. polecki E. nana I. bütschlii E. gingivalis N. fowleri Acanthamoeba
Size 8-22 μm 5-12 μm 8-35 μm 10-20 μm 4-12 μm 5-22 μm - - 8-25 μm
Shape Spherical to Spherical to Round to spherical Spherical or oval Spherical, ovoid, Ovoid, ellipsoid, - - Roundish with ragged
round round ellipsoid triangular, edges
other shapes
No. of nuclei 1-4 1-4 1-8 1 1-4; 4 most 1 - - 1
common
Karyosome Small and central Small and central Large, irregular Small and central Large, blotlike, Large, eccentric - - Large and central
shape, eccentric usually achromatic
central granules on one
side may
be present
Peripheral Fine and evenly Fine and evenly Unevenly Fine and evenly Absent Absent - - Absent
Chromatin distributed distributed distributed distributed
Cytoplasm Finely granular Finely granular Coarse and Granular Granular and Coarsely granular - - Disorganized,
granulated vacuolated and granular,
vacuolated sometimes
vacuolated
Cytoplasmic Chromatoid bars, Chromatoid bars, Diffuse glycogen Chromatoid bars, Chromatin granules Well-defined - - Double cell wall—
inclusions rounded rounded mass present in angular or pointed Nondescript small glycogen mass smooth
ends in young ends in young young cysts; may ends in young cysts mass Granules may be inner cell wall and
cysts cysts displace nuclei Glycogen mass in Diffuse glycogen present outer
Diffuse glycogen Diffuse glycogen (often seen in cysts young cysts mass in young cysts jag ged cell wall
mass in young mass in young with two nuclei) to Inclusion mass
cysts cysts opposite ends of
the cyst
Thin chromatoid
bars with pointed
to splintered ends
in young cysts

Pictures

PARASITOLOGY RMT SOON! <3 ADM


3
CHAPTER 4: FLAGELLATES TROPHOZOITE
Parameter G. intestinalis C. mesnili D. fragilis T. hominis E. hominis R. intestinalis T. tenax T. vaginalis
Size range 8-20 μm long 5-25 μm long 5-18 μm 7-20 μm long 3-10 μm long 3-7 μm long 5-14 μm long Up to 30 μm long
5-16 μm wide 5-10 μm wide 5-18 μm wide 3-7 μm wide 5-6 μm wide
Shape Pear-shaped, Pear-shaped Irregularly round Pear-shaped Oval; sometimes Ovoid Oval, pear-shaped Ovoid, round or
teardrop half-circle pear-shaped
Motility Falling leaf Stiff, rotary, Progressive, broad Nervous, jerky Jerky Jerky - Rapid, jerky
directional hyaline
pseudopodia
Appearance Bilaterally - - - - - - -
symmetrical
Nuclei Two ovoid-shaped, One with small Two, each One, with a small One with central One, with small One, ovoid nucleus; One, ovoid,
each with a large central or eccentric consisting of masse central karyosome karyosome central karyosome consists of vesicular nondescript
karyosome karyosome clumps of four to No peripheral No peripheral Ring of chromatin region filled with
No peripheral No peripheral eight chromatin chromatin chromatin granules may chromatin granules
chromatin chromatin granules be on nuclear
No peripheral membrane
chromatin

Cytoplasm - - Bacteria-filled - - - - -
vacuoles common

Flagella Four pairs, Four: - Three to five Four total: Two; anterior Five total, all originating All originating
origination of each: Three extending anterior Three directed anteriorly: anteriorly:
One pair, anterior from anterior One posterior anteriorly Four extend anteriorly Three to five
end end extending from the One directed One extends posteriorly extending
One pair, posterior One extending posterior end of posteriorly anteriorly
end posteriorly from the undulating One extending
Two pair, central, cytostome region membrane posteriorly
extending laterally

Other structures Two median bodies Prominent - Axostyle that None Cytostome Undulating membrane Undulating
Two axonemes cytostome extends beyond extending halfway extending two membrane
Sucking disk extending the posterior end down body length thirds of body length with extending half of
1/3 to 1/2 body of the body with accompanying costa body length
length Full body length well-defined fibril Thick axostyle curves Prominent axostyle
Spiral groove undulating border opposite around nucleus; that often curves
membrane the nucleus in the extends beyond body around nucleus;
Conical cytostome anterior end length granules may be
cleft in anterior Small anterior cytostome seen along axostyle
region ventrally opposite
located opposite undulating membrane
the undulating
membrane

PARASITOLOGY RMT SOON! <3 ADM


4

CHAPTER 4: FLAGELLATES CYST


Parameter G. intestinalis C. mesnili D. fragilis T. hominis E. hominis R. intestinalis T. tenax T. vaginalis
Size range 8-17 μm long 5-10 μm long - - 3-10 μm long 3-9 μm long - -
6-10 μm wide 4-7 μm wide up to 5 μm wide
Shape Ovoid Lemon-shaped, - - Oval, elongated Lemon-shaped, - -
with a clear hyaline pear-shaped
knob extending
from the anterior
end
Nuclei Immature cyst, two One, with large - - One to four One, located in - -
mature cyst, four central karyosome Binucleated and anterior-central
central karyosomes No peripheral quadrinucleated region w/ central
No peripheral chromatin nuclei located at karyosome
chromatin opposite ends May be surrounded
Central karyosome by a
No peripheral delicate ring of
chromatin chromatin granules

Cytoplasm Retracted from cell - - - - - - -


wall
Other structures Median bodies: two Well-defined - - None Two fused fibrils - -
in immature cytostome located resembling a
cyst or four in fully on one side of the bird’s beak in the
mature cyst nucleus anterior nuclear
Interior flagellar region, only visible
structures* in stained
preparations
*Twice as many in mature cyst as compared with immature cyst.

PARASITOLOGY RMT SOON! <3 ADM


5

CHAPTER 5: THE HEMOFLAGELLATES


Amastigote Promastigote Epimastigote Trypomastigote
Size 5 by 3 µm 9-15 µm long 9-15 µm long 12-35 µm long by 2-4 µm wide
Shape Round to oval Long and slender Long and slightly wider than promastigote form C, S or U shape often seen in stained blood
films
Appearance - - - Long and slender
Nucleus One, usually off center One, located in or near center One, located in posterior end One, located anterior to the kinetoplast
Other features Kinetoplast present, Kinetoplast, located in anterior end Kinetoplast located anterior to the nucleus Kinetoplast located in the posterior end
consisting of dotlike Single free flagellum, extending from Undulating membrane, extending half of body Undulating membrane, extending entire body
blepharoplast from which anterior end length length
emerges a small axoneme Free flagellum, extending from anterior end Free flagellum, extending from anterior end
Parabasal body located when present
adjacent to the
blepharoplast
Pictures

PARASITOLOGY RMT SOON! <3 ADM


6

CHAPTER 6: PLASMODIUM AND BABESIA


Plasmodium vivax Plasmodium ovale Plasmodium malariae Plasmodium falciparum Plasmodium
knowlesi
Relative age of Only young and immature cells Only young and immature cells Only mature cells Infect cells of all ages
-
infected RBCs
Appearance of Enlarged, distorted Oval and enlarged, distorted with Normal size, no distortion Normal size, no distortion
-
infected RBCs ragged cell walls
Morphologic Form -
Ring form Delicate cytoplasmic ring measuring one third Resembles P. vivax Smaller than P. vivax Circle configuration or
of RBC diameter Ring larger in size than P. vivax Occupies one sixth of the RBC headphone configuration
Single chromatin dot Ring thick and often somewhat Heavy chromatin dot Scanty cytoplasm
-
Ring surrounds a vacuole ameboid in appearance Vacuole may appear filled in Small vacuole usually visible
Accolé forms possible Pigment characteristically forms early Multiple rings common
Accolé forms possible
Developing Irregular ameboid appearance Ring appearance usually Nonameboid solid cytoplasm that may Heavy rings common
trophozoite Ring remnants common Brown pigment maintained until late in assume roundish, oval, band, or bar shape Fine pigment granules
becomes apparent, increases in number and development Cytoplasm contains coarse dark brown Mature forms only seen in -
visibility as parasites mature Ameboid tendencies not as evident pigment; may mask chromatin material severe infections
as in P. vivax Vacuoles absent in mature stages
Immature schizont Multiple chromatin bodies Progressive dividing chromatin Similar to that of P. vivax, only smaller; may Multiple chromatin bodies
Often contains clumps of brown pigment surrounded by cytoplasmic contain large and dark peripheral or central surrounded by cytoplasm
-
material—often maintains circular granules Only detected in severe
shape early in development infections
Mature schizont 12 to 24 merozoites occupy most of infected Parasites occupy 75% of RBCs. Typically contains 6 to 12 merozoites Typically consists of 8-36
red blood cell Rosette arrangement of merozoites arranged in rosettes or irregular clusters merozoites (average, 24) in
Merozoites surrounded by cytoplasmic material (average of eight merozoites Central arrangement of brown-green pigment cluster arrangement
Brown pigment may be present typically present) may be visible Only detected in severe -
Infected RBC may not be seen because infections
developing parasites often fill the cell
completely.
Microgametocyte Large pink to purple chromatin mass Similar to P. vivax, only smaller in Similar to P. vivax, only smaller in size; Sausage- or crescent-shaped
surrounded by colorless to pale halo size pigment usually darker and coarser Dispersed central chromatin
-
Brown pigment common Older forms assume an oval shape with nearby black pigment
usually visible
Macrogametocyte Round to oval cytoplasm Sausage- or crescent-shaped
Eccentric chromatin mass Compact chromatin
-
Delicate light-brown pigment— may be visible Black pigment surrounding
throughout cell chromatin may be visible

CHAPTER 6: PLASMODIUM AND BABESIA CONTINUATION…


Babesia Species Trophozoite Babesia Species Merozoite
Appearance Resembles a ring form Resembles four trophozoites attached by their respective
Does not contain Schüffner’s, Ziemann’s, or Maurer’s dots chromatin dots in the shape of a Maltese cross
Ring characteristics when Blue cytoplasmic circle connected with or to red chromatin dot
stained with Giemsa Vacuole usually present

PARASITOLOGY RMT SOON! <3 ADM


7

CHAPTER 7: MISCELLANOUS PROTOZA


Trophozoites Cyst Oocyst Vacuolated forms Spores Tachyzoites Bradyzoites
Balantidium Coli • largest Size range: 43-66
protozoan μm
known to Number of nuclei:
humans • Two Kidney-
Size range: 28- shaped
152 μm in length, macronucleus
22-123 μm wide • Small spherical
Motility: Rotary, micronucleus;
boring may not be
Number of observable
nuclei: Other features:
• Two kidney- • One or two
shaped visible
macronucleus contractile
• Small vacuoles in
spherical young cysts
- - - - -
micronucleus • Double cyst
Other features: wall
• One or two Row of cilia visible
visible in between cyst
contractile wall layers of
vacuoles young cysts
• Cytoplasm
may contain
food vacuoles
and/or
bacteria
• Small
cytostome
present
Layer of cilia
around organism
Isospora belli - - Size range: 25-35 μm long, - - - -
10-15 μm wide
Appearance: Transparent
Shape: Oval
Cell wall: Two layered,
colorless and smooth
Developing sporoblast:
Unicellular with granular
cytoplasm
Young oocyst: Two
sporoblasts

PARASITOLOGY RMT SOON! <3 ADM


8

Mature oocyst: Two


sporocysts, each containing
four sausage-shaped
sporozoites
Sarcocystis - - Shape: oval - - - -
species Appearance: transparent
No. of sporocysts: two
Size of each sporocysts:
10-18 µm long
Contents of each
sporocysts: Four sausage-
shaped sporozoites
Oocyst cell wall
appearance: Clear,
colorless, double layered
Cryptosporidium - - Size: 4-6um - - - -
parvum Shape: roundish
No. of sporocysts: none
No. of sporozoites: four
(small)
Other features:
• Thick cell wall One to six
dark granules may be
visible
Blastocystis - - Size: 5 to 32 µm - - -
hominis Vacuole:
• Centrally located
Fluid-filled structure
Consumes almost
90% of organism
Cytoplasm
• Appears as ring
around periphery of
organism
Nuclei
• Two to four located in
cytoplasm
Cyclospora - - Size: 7-10 µm in diameter - - - -
cayetanensis Number of sporocysts: two
Contents of sporocysts:
Each sporocyst contains
two sporozoites
Microsporidia - - - - Size: 1-5 µm - -
Other features:
• Equipped with
extruding polar

PARASITOLOGY RMT SOON! <3 ADM


9

filaments (or tubules)


that initiate infection
by injecting
sporoplasm
(infectious material)
into host cell
Toxoplasma • Infective form is oocyst General comment: General comment: Slow-
gondii • Similar in appearance Actively multiplying growing morphologic form
to I. belli morphologic form Size: Smaller than tachyzoites
• T. gondii is smaller Size: 3-7 × 2-4 µm Physical Apperance:
• Round to slightly oval Shape: Crescent- Crescent-shaped, often more
• 10 to 15 µm long by 8 shaped, often more rounded on one end
to 12 µm wide rounded on one Other features: Hundreds to
• Transparent oocyst – 2 end thousands of bradyzoites
sporocysts, each has 4 Number of nuclei: enclose themselves to form a
sporozoites One cyst that may measure 12-100
• clear, colorless, two- Other features: µm in diameter
layered cell wall Contains a variety
of organelles that
are not readily
visible
Pneumocystis Size: 2-4 µm Size: Diameter, 4- - - - - -
jiroveci Shape: Ovoid, 12 µm
ameboid Shape: Roundish
Number of Number of nuclei:
nuclei: one Four to eight;
unorganized or
arranged in a
rosette

PARASITOLOGY RMT SOON! <3 ADM


10

CHAPTER 8: NEMATODES

Enterobius vermicularis Trichuris trichiura Ascaris lumbricoides Hookworms Strongyloides stercoralis


Eggs Size: 48-60 µm long, 20-35 µm wide Size: 50-55 by 25 µm Size: 85-95 µm by 38-45 µm; size Size Size: Average, 48 by 35 µm
Shape: Oval, one-side flattened Shape: Barrel, football; hyaline polar variations possible Length: Typical growth phase: Contains
Embryo: Stage of development varies; plug at each end Shape: Varies Necator, 60-75 µm; well-developed larvae
may be unembryonated, embryonated, Embryo: Unicellular; undeveloped Embryo: Unembryonated; Ancylostoma, 55-60 µm Embryonic cleavage: Two-,
mature Shell: Smooth; yellow-brown color amorphous mass of protoplasm Width: 35-40 µm four-, or eight-cell stage, when
Shell: Double-layered, thick, colorless because of bile contact Shell: Thin Embryonic cleavage: Two-, present
Other features: Usually corticated four-, or eight-cell stage Shell: Thin, hyaline
Shell: Smooth, colorless

Adults Female: Size: 2.5-5 cm long; males usually Female: Female: Female:
Length: 7-14 mm smaller than females Size: 22-35 cm Size: 9-12 mm long, 0.25-0.50 Approximate size: 2 by 0.4 mm
Width: Up to 0.5 mm Anterior end: Color: Creamy white pink tint mm wide Other features: Colorless,
Color: Yellowish-white Colorless; resembles a whip handle; Other features: Pencil lead Male: transparent body; finely
Tail: Pointed; contains a slender esophagus thickness Size: 5-10 mm long, 0.2-0.4 mm striated cuticle; short buccal
resembles pinhead Posterior end: Male: wide cavity; Long and slender
Male: Pinkish-gray; resembles whip itself; Size: Up to 30 cm Other features: Prominent esophagus
Length: 2-4 mm contains digestive and reproductive Color: Creamy white pink tint posterior copulatory bursa
Width: ≤0.3 mm systems; males possess prominent Other features Prominent General Characteristics
Color: Yellowish-white curled tail incurved tail Color: Grayish-white to pink
* Rarely seen Cuticle: Somewhat thick
Anterior end: Conspicuous
bend, hook

Buccal Capsule Characteristics:


Necator: Contains pair of
cutting plates

Ancylostoma: Contains actual


teeth

PARASITOLOGY RMT SOON! <3 ADM


11

Rhabditiform Size Average size: 220 by 15 µm


larva Newly hatched: 270 by 15 µm Other features: Short buccal
5 days old: 540-700 µm long cavity; prominent genital
Other features: Long buccal cavity;
primordium
small genital primordium

Filariform - - - Length of esophagus: short Average length: 690 µm


Larva Tail: pointed Length of esophagus: Long
Tail: notch

PARASITOLOGY RMT SOON! <3 ADM


12

CONTINUATION OF NEMATODES
Strongyloides stercoralis Trichinella spiralis Trichuris trichiura Ascaris lumbricoides
Eggs Size: Average, 48 by 35 µm - Size: 50-55 by 25 µm Size: 85-95 µm by 38-45 µm; size variations
Typical growth phase: Contains well- Shape: Barrel, football; hyaline polar plug at possible
developed larvae each end Shape: Varies
Embryonic cleavage: Two-, four-, or eight- Embryo: Unicellular; undeveloped Embryo: Unembryonated; amorphous mass of
cell stage, when present Shell: Smooth; yellow-brown color because protoplasm
Shell: Thin, hyaline of bile contact Shell: Thin
Other features: Usually corticated

Adults Female: Female: Size: 2.5-5 cm long; males usually smaller Female:
Approximate size: 2 by 0.4 mm Size: 4 by 0.5 mm than females Size: 22-35 cm
Other features: Colorless, transparent body; Notable features: Blunt, round posterior Female: Color: Creamy white pink tint Other features:
finely striated cuticle; short buccal cavity; end; single ovary with vulva in anterior Size: 840 by 1.5 mm Pencil lead thickness
Long and slender esophagus fifth of body Other features: Prominent rounded anterior Male:
end Size: Up to 30 cm
Male: Color: Creamy white pink tint
Size: 2 by 0.04 mm Male: Other features: Prominent incurved tail
Notable features: Curved posterior end Size: 21 by 0.4 mm
with two rounded appendages Other features: Anterior end coils itself at
least once
Common to both male and female adult:
Thin anterior end
Small mouth
Long slender digestive tract

Rhabditiform - - Average size: 620 by 15 µm -


larva Tail characteristics: Consumes one third of
body length; culminates in a point
Filariform Average size: 220 by 15 µm - - -
Larva Other features: Short buccal cavity;
prominent genital primordium

PARASITOLOGY RMT SOON! <3 ADM


13

Encysted Average length: 690 µm Average juvenile size: 75-120 µm long, 4- - -


larva Length of esophagus: Long 7 µm wide
Tail: notch Average mature size: Up to 1 mm in
length
Appearance: Colied
Encysted in: Nurse cells of striated
muscle
Notable features: Inflammatory infiltrate
present around nurse cell

PARASITOLOGY RMT SOON! <3 ADM


14

CHAPTER 9: THE FILARIAE


Microfilariae Adult
Wuchereria bancrofti Size range: 240-300 µm long Sheath: Present • White
Arrangement of nuclei in tail: Tip of tail free of nuclei • threadlike appearance
• females (40 to 100 mm) are larger than
the males (20 to 40 mm)

Brugia malayi Size range: 200-280 µm long Sheath: Present • resemble those of W. bancrofti
Arrangement of nuclei in tail: Presence of two distinct nuclei in the tip of the tail; the organism tissue • White
tends to bulge around each of the two nuclei • threadlike appearance
• female: 54mm length
• male: 24 mm length

Loa loa Size range: 248-300 µm long Sheath: Present • White


Arrangement of nuclei in tail: Distinct continuous row of nuclei; extend to tip of tail • Cylindrical threadlike appearance
• female: 38 to 72 mm length
• male: 28 to 35 mm length

Onchocerca volvulus Size range: 150-355 µm long Sheath: Absent • thin


Arrangement of nuclei in tail: Do not extend to tip of tail • wirelike appearance
• coil up in knots
• female: up to 500 mm length
• male: 25 to 50 mm length

Mansonella ozzardi Size range: 220 µm in length Sheath: Absent • female: 65 to 80 mm length
Arrangement of nuclei in tail: Numerous; Do not extend to tip of tail • male: 32 mm length
• location in humans is currently unknown

Mansonella perstans Size range: about 200 µm in length Sheath: Absent • female: 82 mm in length
Arrangement of nuclei in tail: Numerous; Extend to tip of tail • male: 43 mm in length.
• peritoneal and pleural cavities as well as
the mesentery

PARASITOLOGY RMT SOON! <3 ADM


15

CHAPTER 11: THE CESTODES


Eggs Adult Pictures
Taenia Size range: 28-40 µm by 18-30 µm Characteristic T. saginata T. solium
Hooklets: Three pairs; hexacanth embryo
Other features: Radial striations on yellowbrown Scolex
embryophore Number of suckers Four Four
Rostellum Absent Present
Hooks Absent Present: double
crown

Gravid Proglottid Longer than


Appearance, shape wide; average, Somewhat square
17.5 by 5.5 µm

Number of lateral 15-30 7-15


branches on each side of
uterus

Hymenolepis Average size: 55 by 85 µm Scolex Gravid Proglottid


diminuta Hooklets: Three pairs; hexacanth embryo Number of suckers: Four Size: Twice as wide as long
Polar thickenings: Present Rostellum: Present Appearance: Saclike uterus filled
Polar filaments: Absent Hooks: Absent with eggs
Embryophore: Present; colorless
Hymenolepis nana Average size: 45 by 38 µm Scolex Gravid Proglottid
Hooklets: Three pairs; hexacanth embryo Number of suckers: Four Shape: Twice as wide as long
Polar thickenings: Present Rostellum: Present; short Appearance: Saclike uterus filled
Polar filaments: Present Hooks: Absent with eggs
Embryophore: Present; colorless
Dipylidium caninum Number of eggs in enclosed packet: 5-30 Scolex Gravid Proglottid
Diameter range per egg: 30-60 µm Number of suckers: Four Shape: Pumpkin seed
Individual egg features: Six-hooked oncosphere Rostellum: Present; club- Appearance: Full of eggs in
shaped, with one to seven enclosed embryonic membrane
circlets of spines
Hooks: Absent
Diphyllobothrium Size range: 55-75 µm long, 40-55 µm wide Scolex Gravid Proglottid
latum Shape: Somewhat oblong Number of suckers: Four Shape: Pumpkin seed
Embryo: Undeveloped, termed coracidium Rostellum: Present; club- Appearance: Full of eggs in
Shell: Smooth; yellow-brown in color shaped, with one to seven enclosed embryonic membrane
Other features: Operculum on one end; circlets of spines
terminal knob on opposite end Hooks: Absent
Echinococcus HYDATID CYST • 4.5 mm in length
granulosus Protective coverings: Cyst wall; multiple • scolex, small neck, and three proglottids, one at each
laminated germinal tissue layers developmental stage—immature, mature, and gravid.
Basic cyst makeup: Fluid-filled bladder • Scolex – four suckers with 36 hooks
Structures that arise from inner germinal layer: - Found in canines (definitive host)
Daughter cysts, Brood capsules
Other possible structures present: Hydatid sand

PARASITOLOGY RMT SOON! <3 ADM


16

CHAPTER 11: THE TREMATODES


Eggs Adults Pictures
Fasciolopsis buski Size range: F. buski, 128-140 µm by 78-85 µm; • F. buski - 5 by 1.5 cm
& Fasciola hepatica F. hepatica, 128-150 µm by 60-90 µm • F. hepatica - 3 by 1 cm;
Shape: Somewhat oblong equipped with
Egg contents: Undeveloped miracidium “shoulders”
Other features: Presence of a distinct operculum

Clonorchis sinensis Size range: 30 by 15 µm • 2 by 0.5 cm


Egg contents: Developed miracidium • Each end of the adult
Operculum: Present worm is narrower than
Other features: Presence of distinct shoulders and presence the midportion of the
of small knob opposite operculum body
Heterophyes heterophyes Size range: 30 by 15 µm Heterophyes:
& Metagonimus Egg contents: Developing miracidium • 1.0 by 0.5 mm
yokogawai Operculum: Present • pyriform in shape
Shoulders: Present but discrete • grayish in color
Small knob: May be absent • protected by an outer layer of fine spines – “scaly” appearance
Shell thickness: Heterophyes, thick; Metagonimus, thin Metagonimus:
• 1.5 by 0.5 mm
• pyriform in shape
• anterior – tapered
• posterior – round
• tiny layer of scaly spines
Paragonimus westermani Size range: 78-120 µm long; 45-60 µm wide • somewhat oval
Shape: Somewhat oval • red- to brown-colored
Egg contents: Undeveloped miracidium surrounded by a • 1 by 0.7 cm
thin, smooth shell • Spines
Other features: Prominent operculum with shoulders;
obvious terminal shell thickening opposite operculum
Schistosoma mansoni Size range: 112-182 µm by 40-75 µm • have separate sexes
Shape: Oblong • round
Egg contents: Developed miracidium • female - 2 cm in length
Other features: Large; lateral • male surrounds the
female almost completely
Schistosoma japonicum Size range: 50-85 µm by 38-60 µm
Shape: Somewhat oblong
Egg contents: Developed miracidium
Other features: Small; lateral

PARASITOLOGY RMT SOON! <3 ADM


17

Schistosoma haematobium Size range: 110-170 µm by 38-70 µm


Shape: Somewhat oblong
Egg contents: Developed miracidium
Other features: Large; terminal

CHAPTER 13: THE ARTHROPODS


Adult
Ticks Characteristic Hard Ticks Soft Ticks
Fused spherical body
(head, thorax, abdomen together) X X
Four pairs of legs X X
Visible capitulum on dorsal side X
Capitulum on ventral side X
Scutum X
Mites Size: 0.1-0.4 mm
Shape: Oval

Spiders Black widow spiders • Males and females have shiny black surface
• Females larger than males
• Females have diagnostic red hourglass shape on underside
• Males and females spin atypical or chaotic webs
• Males and females most active at night
Brown recluse • Exhibit reclusive behavior
spiders • Often hide in clothing or bedding
• Only bite when threatened
• Have trademark brown violin shape seen on the cephalothorax
Hobo spiders • Have muted, nondistinct, brown earth color
• Spin distinctive funnel-shaped web
Scorpions Body: Combined cephalothorax and elongated abdominal section that ends in up-curved tail
Final segment in tail contains stinger and venom sac
Legs: Four pairs; first set is enlarged, with clawlike pincers to hold prey
Fleas Size: 1.3-4 mm
Mouth parts: Designed for piercing and blood sucking
Body: Compact and segmented
Legs: Three pairs; hairy with clawlike feet, back pair long for jumping
Flies Size: 1-15 mm long Mouth parts: Presence or absence of hairs
Eyes: One pair Number of segments:
Antennae: One pair Antennae: Structures present for piercing of skin
Legs: Usually three pairs or sucking
Wings: Two pairs, one pair smaller Body: Size and shape as a whole
Body: Three separate sections— Size and shape of head, thorax, and
head, thorax, and segmented abdomen abdomen independent

PARASITOLOGY RMT SOON! <3 ADM


18

Color and distribution of hair


Wings: Pattern of veins
Lice Size: Head and body lice, 2-3 mm long
Crab lice, up to 2 mm long
Mouth parts: Adapted for piercing human skin and sucking blood
Body: Head and body lice, three segments—narrow head, thorax, abdomen
Crab lice—plump; thorax and abdomen appear as one section
Legs: Three pairs extending from thorax with clawlike feet
Antennae: One pair
Hair: Head and body lice—absent Crab lice—present on extremities
Mosquito Body: Three segmented body consisting of head, thorax and abdomen
Head: Roundish in shape
Neck: Slender; connects head and thorax
Thorax: Elongate
Abdomen: Elongate, comprised of ten segments
Antennae: One pair: each is long, with three segments
Legs: Three pairs, extending from thorax region
Wings: Two pairs, one smaller than the other
Bugs Appearance: Varies; bedbugs have a reddish-brown outer shell
Size: Varies; bedbugs may be up to 5 mm, whereas triatomid bugs are usually up to 4 cm
in length
Legs: Three pairs; triatomid bugs and select cockroaches have fully developed wings in the
adult stage
Wings: At some point in the life cycle; some wings may be vestigial

PARASITOLOGY RMT SOON! <3 ADM


1
CHAPTER 3: THE AMOEBAS
Entamoeba histolytica Entamoeba hartmanni Entamoeba coli Entamoeba polecki Endolimax nana
Intestinal amebiasis, amebic colitis, Intestinal amebiasis, amebic Intestinal amebiasis, None (considered as a None (considered as a
Common associated amebic dysentery, extraintestinal colitis, amebic dysentery, amebic colitis, amebic nonpathogen) nonpathogen)
disease amebiasis extraintestinal amebiasis dysentery, extraintestinal
amebiasis
Laboratory Diagnosis • Stool - wet preparation and • stool - trophozoites and cysts • Stool examination is the • stool examination • stool examination
permanent staining • size ranges of E. histolytica method of choice
• TYI-S-33 – medium for culture and E. hartmanni overlap • ingestion of
• ELISA, IHA, GDP, IIF • micrometer is essential contaminated food or
drink
Life Cycle Notes • infective cyst is ingested - excystation • • • •
in the small intestine
• Lumen of the large intestine – trophs
replicate by binary fission
- Encystation
- Complete = four nuclei is present
• Cysts - normal intestinal motility
• trophozoites - rapid intestinal motility
Epidemiology • Subtropical, tropical areas & colder • cosmopolitan distribution • Worldwide • parasite of pigs and • warm, moist regions
climates • similar to that of E. • warm & cold climates monkeys • areas w/ poor hygiene and
• prevalent in homosexuals histolytica in areas • areas w/ poor hygiene • highest prevalence substandard sanitary
• hand-to-mouth contamination • ingestion of infected cysts and sanitation practices – occurring in Papua, New conditions
• unprotected sex can be endemic Guinea • ingestion of infective cysts in
• vectors - flies and cockroaches • Ingestion of the E. polecki contaminated food and
cyst drinks
Clinical Symptoms • only known pathogenic intestinal • asymptomatic • asymptomatic • asymptomatic • asymptomatic
ameba • symptomatic patients is
• range of symptoms depends on: diarrhea
- location(s) of the parasite in the host
- extent of tissue invasion
• Asymptomatic
- low-virulence strain
- inoculation into the host is low
- patient’s immune system is intact
• Intestinal Amebiasis
- amebic colitis
- amebic dysentery – blood and
mucus in stool
- Secondary bacterial infections
- flask-shaped amebic ulcers
• Extraintestinal Amebiasis
- Liver
- amebic pneumonitis
- lung, pericardium, spleen, skin,
and brain

PARASITOLOGY RMT SOON! <3 ADM


2
- Venereal amebiasis
Treatment • Asymptomatic: paromomycin, • nonpathogen and treatment • nonpathogen and • combination of • nonpathogen and treatment
diloxanide furoate (Furamide), or is usually not indicated treatment is usually not metronidazole (Flagyl) and is usually not indicated
metronidazole (Flagyl) indicated diloxanide furoate
• intestinal amebiasis: iodoquinol, (Furamide)
paromomycin, or diloxanide furoate • Metronidazole
• extraintestinal amebiasis:
Metronidazole or tinidazole, in
combination with a symptomatic
intestinal amebiasis treatment
Prevention and Control • Uncontaminated water - boiling or • Good sanitation • adequate disposal of • improving personal • protection of food and drink
treating with iodine crystals • personal hygiene practices human feces hygiene and sanitation from flies and cockroaches
• infective (quadrinucleated) cyst – • protection of food from flies • proper personal hygiene practices • good sanitation and personal
resistant to chlorination and cockroaches practices • Education programs hygiene practices
• water treatment regimen • Protection of food and
• Properly washing food products drink from flies and
• avoiding the use of human feces as cockroaches
fertilizer
• good personal hygienek
Side Notes: • karyosome - small central mass of • “small race” E. histolytica
chromatin
• peripheral chromatin - chromatin
material that surrounds the karyosome
• chromatioid bars -squared or
roundended structures
• glycogen mass - stored food
• Invasive and noninvasive strains of E.
histolytica may be distinguished by
performing isoenzyme electrophoresis
and examining the zymodemes
(isoenzyme patterns)
• WHO – O&P examination obsolete
• Entamoeba dispar - morphologically
identical to E. histolytica
• E. histolytica - contain ingested RBCs

PARASITOLOGY RMT SOON! <3 ADM


3
Iodamoeba bütschlii Entamoeba gingivalis Naegleria fowleri Acanthamoeba
None (considered as a None (considered as a Primary amebic meningoencephalitis Granulomatous amebic encephalitis (GAE),
Common associated disease
nonpathogen) nonpathogen) (PAM) Acanthamoeba keratitis
Laboratory Diagnosis • stool examination • examining mouth scrapings • Microscopic examination of • CSF – spx of choice
• Iodine wet preps – cysts, • tonsillar crypts, pulmonary cerebrospinal fluid - best • Brain tissue, corneal scrapings – for eye
glycogen mass abscess, as well as sputum, • saline and iodine wet preparations of infections
• glycogen mass unstained vaginal and cervical material the CSF • tracks - set of marks on the agar
after trichrome staining may be examined • trophs - trailing effect when placed on • IIF antibody staining – choice for speciation
agar plates that have been previously
inoculated with gram-negative bacilli
Life Cycle Notes • • lives around the gum line of the • Trophs – only form to exist in humans • aspiration or nasal inhalation
teeth in the tartar and gingival • ameboid trophozoites transform into - enter via the lower respiratory tract or
pockets of unhealthy mouths through ulcers in the mucosa or skin
flagellate trophozoites in vitro after
• trophozoites - inhabit tonsillar
being transferred to water from a tissue - hematogenous spread to CNS
crypts and bronchial mucus
• E. gingivalis and E. histolytica –
or culture • direct invasion in the eye
both found in the sputum and in • flagellate trophozoites do not divide but - contact lens wearers
pulmonary abscesses rather lose their flagella and convert - trauma to the cornea
• feed on disintegrated cells back into the ameboid form - Acanthamoeba keratitis
• Spontaneous disappearance after • swimming in contaminated water - contaminated saline
removal of IUD
Epidemiology • higher prevalence in tropical • mouth-to-mouth (kissing) and • warm bodies of water • worldwide
regions than in temperate droplet contamination • Prevalence is higher in the summer • CNS infections - debilitated or
regions months immunocompromised
• contaminated dust
• ameboid trophs enter the nasal mucosa
Clinical Symptoms • asymptomatic • asymptomatic • Asymptomatic • Granulomatous amebic encephalitis (GAE)
• Primary amebic meningoencephalitis: - stiff neck
- Invade the brain, rapid tissue destruction - Granulomatous lesions of the brain
- stiff neck and seizures
• Acanthamoeba keratitis
- Kernig’s sign - diagnostic sign for
meningitis - hamstring stiffness - severe ocular pain and vision
- perforation of the cornea -> loss of
vision
Treatment • nonpathogen and treatment • nonpathogen and treatment is • prompt and aggressive treatment with • slow progression -> die
is usually not indicated usually not indicated amphotericin B – toxic but effective • sulfamethazine
• amphotericin B in combination with rifampin • Acanthamoeba keratitis - itraconazole,
or miconazole – for rare cases ketoconazole, miconazole, propamidine
isethionate (BEST), and rifampin
Prevention and Control • personal hygiene and • Improved oral hygiene • Posting off-limits signs around known sources • following all manufacturer-established
sanitation practices • Prompt removal of IUDs of contamination protocols associated with the use of
• Education contact lenses
• swimming pools and hot tubs be adequately
chlorinated
Side Notes: • Iodamoeba - ameba that stains • Discovered in 1849 • first case of PAM was reported by Carter and • shares many characteristics with the gram-
well with iodine • first ameba recovered from a Fowleri in Australia, 1965; Butt and Patras in negative bacteria Pseudomonas aeruginosa
• Nucleus - resembling a basket of human specimen 1966 in the United States
flowers • N. australiensis - possibly infect humans in the
future

PARASITOLOGY RMT SOON! <3 ADM


4

NOTES:
• pseudopods - false feet – for movement
• two morphologic forms
o trophozoites - feeds, multiplies, and possesses pseudopods; delicate and fragile
o cyst - nonfeeding stage, thick protective cell wall
• Excystation - from the cyst form into the trophozoite form
• Encystation - conversion of trophozoites to cysts
• Saline wet preparations - show motility of the amebic trophozoites
• iodine wet preparations – show internal cytoplasmic & nuclear structures
• permanent smear procedure - shrink amebic parasites

PARASITOLOGY RMT SOON! <3 ADM


5
CHAPTER 4: FLAGELLATES
Giardia intestinalis Chilomastix mesnili Dientamoeba fragilis Trichomonas hominis
Common Giardiasis, traveler’s diarrhea None (considered a nonpathogen) Dientamoeba fragilis infection None (considered as a nonpathogen)
associated disease
Laboratory • stool • Liquid stool – trophs • Examination of stool • Stool examination
Diagnosis • multiple samples – recommended • Formed stool - cysts • Multiple samples
• Enterotest • Encystation in unformed samples • ability to blend in well with the
• EIA, ELISA • Iodine wet preparations - background material
• Direct Fluorescence - Giardia and Cryptosporidium demonstrate the organism’s • RT-PCR – for diagnosis
features most clearly
Life Cycle Notes • infective cysts enter the stomach – excyst • not well understood
• encystation -> trophs migrate into the large bowel • resides in the mucosal crypts of the
• cyst - viable for as long as 3 months in water large intestine
• ingest red blood cells
Epidemiology • Worldwide • prefers warm climates • exact mode of transmission – unknown • warm climates
• Cysts - resistant to the routine chlorination • personal hygiene and poor sanitary • theory - D. fragilis is transmitted via the • children appear to contract more
• Person-to-person contact conditions prevail are at the eggs of Enterobius vermicularis and often than adults
• fecal-oral route greatest risk Ascaris lumbricoides • achlorhydria – milk acts as shield
• unprotected sex – homosexuals • ingestion of infective cysts • at risk -> children, homosexual men, for trophs upon entry to stomach
• reservoir hosts - beavers, muskrats, and water semicommunal groups, institutionalized • Fecal-oral transmission
voles • fecal-oral, oral-anal routes, person-to-
person route
Clinical Symptoms • Asymptomatic • asymptomatic • Asymptomatic • asymptomatic
• Giardiasis • Symptomatic
- light-colored stools with a high fat content - diarrhea and abdominal pain, low-
- foul-smelling diarrhea grade eosinophilia, and pruritus
• incubation period is 10 to 36 days
• self-limiting (10 to 14 days)
• px w/ intestinal diverticuli or IgA deficiency -
reoccurring infections
Treatment • Metronidazole, tinidazole, and nitazoxanide • Treatment not indicated - • Iodoquinol – BEST • Treatment not indicated -
• Metronidazole, tinidazole – effective but nonpathogen • Paromomycin – if treatments is not nonpathogen
carcinogenic appropriate
• Nitazoxanide – approved by FDA
Prevention and • Proper water treatment • Proper personal hygiene • maintaining personal • Improved personal
Control • Double-strength saturated iodine solution added • public sanitation practices • public sanitary conditions • public sanitary practices
to water before drinking • avoidance of unprotected homosexual
practices
Side Notes: • Cercomonas intestinalis, Giardia duodenale • initially classified as an ameba
• 1859 by French scientist Dr. F. Lambl & • D. fragilis returns to its normal size – in
Czechoslovakian scientist Dr. Giard water preparations
• discovered in 1681 by Anton van Leeuwenhoek • Brownian motion
• trophs – “old man with whiskers”, “monkey face”, • Hakansson phenomenon
“cartoon character”
• G. intestinalis and Trichomonas vaginalis - carriers
of doublestranded RNA viruses

PARASITOLOGY RMT SOON! <3 ADM


6

CHAPTER 4: FLAGELLATES CONTINUATION…


Enteromonas hominis Retortamonas intestinalis Trichomonas tenax Trichomonas vaginalis
Common associated None (considered as a None (considered as a None (considered as a Persistent urethritis, persistent vaginitis, infant Trichomonas vaginalis
disease nonpathogen) nonpathogen) nonpathogen) infection
Laboratory Diagnosis • Examination of stool • stained stool preparation • mouth scrapings, tonsillar crypts • spun urine, vaginal discharges, urethral discharges, and prostatic
• difficult to identify accurately • difficult to identify and pyorrheal pockets secretions
because of its small size accurately because of its • Tartar between the teeth and • saline wet preparations – preferred, motility, short amount of
small size gingival margin of the gums - time
harbor this organism • phase contrast microscopy, Papanicolaou (Pap) smears,
fluorescent stains, monoclonal antibody assays, enzyme
immunoassays, and cultures
• DNA-based assay - Affirm VPIII – T. vaginalis detection
• InPouch TV culture system - incubation time and takes up to 3
days before a result
Life Cycle Notes • mouth scavengers • reside on the mucosal surface of the vagina
• multiply by longitudinal binary • multiply by longitudinal binary fission
fission • thrive in a slightly alkaline or slightly acidic pH environment
• unable to survive the digestive • males - prostate gland region and the epithelium of the urethra
process
Epidemiology • worldwide in warm and • warm and temperate • exact mode of transmission is • worldwide
temperate climates climates unknown • sexual intercourse – primary mode of transmission
• Ingestion of infected cysts • ingestion of the infected • contaminated dishes and • contaminated toilet articles or underclothing
cysts utensils, kissing • survive in water for up to 40 minutes
Clinical Symptoms • asymptomatic • asymptomatic • asymptomatic • Asymptomatic
• Persistent Urethritis
- enlarged tender prostate, dysuria, nocturia, and epididymitis
- release a thin, white, urethral discharge
• Persistent Vaginitis
- foul-smelling, greenish-yellow liquid vaginal discharge after an
incubation period of 4 to 28 days
- Burning, itching, and chafing
- Cystitis
• Infant Infections
- respiratory infection and conjunctivitis
- contracted from mother
Treatment • Treatment not indicated - • Treatment not indicated - • Treatment not indicated – • metronidazole – treatment of choice
nonpathogen nonpathogen nonpathogen
• proper oral hygiene practices
Prevention and • proper personal hygiene • improved personal and • Practicing good oral hygiene is • avoidance of unprotected sex
Control • public sanitation practices public hygiene conditions the most effective method • prompt diagnosis and treatment
Side Notes: • • • connection
• between T. vaginalis infections and cervical carcinoma

PARASITOLOGY RMT SOON! <3 ADM


7

NOTES:
• flagella - whiplike structures; for movement
• All flagellate life cycles consist of the trophozoite form
• Giardia intestinalis – duodenum; only pathogenic intestinal flagellate
• undulating membrane - finlike structure
• axostyle - rodlike support
• Axonemes - interior portions of the flagella
• median bodies - with energy, metabolism, or support

PARASITOLOGY RMT SOON! <3 ADM


8

PARASITOLOGY RMT SOON! <3 ADM


9
CHAPTER 5: THE HEMOFLAGELLATES
Leishmania braziliensis complex Leishmania donovani complex Leishmania mexicana complex Leishmania tropica complex
Mucocutaneous leishmaniasis, chiclero ulcer, Visceral leishmaniasis, kala-azar, New World cutaneous leishmaniasis, Old World cutaneous leishmaniasis, oriental
Common
espundia, forest yaws, pian bois, uta dum dum fever chiclero ulcer, bay sore sores, Delhi boils, Baghdad boils, dry or urban
associated disease
cutaneous leishmaniasis
Laboratory • biopsy of the infected ulcer – spx of • Montenegro skin test - used for • amastigote in Giemsa-stained • Giemsa-stained slides of aspiration of
Diagnosis choice screening large populations at risk preparations of lesion biopsy fluid underneath the ulcer bed
• Giemsa-stained – reveal amastigotes • blood, bone marrow, lymph node material • Culture
• Culture, serologic testing may be used aspirates, and biopsies, sternal • Culture on NNN medium • IFA
• schizodeme analysis - nuclear DNA marrow aspirate • Schizodeme analysis, zymodeme • Schizodeme analysis, zymodeme analysis,
hybridization • IFA, ELISA, DAT, schizodeme analysis, analysis, and nuclear DNA and nuclear DNA hybridization
• zymodeme analysis - isoenzyme patterns zymodeme analysis, and nuclear hybridization
DNA hybridization
Life Cycle Notes • Lutzomyia and Psychodopygus – • identical to that of L. braziliensis • identical to that of L. braziliensis • identical to that of L. braziliensis except
transmit the promastigotes via blood except: • primary vectors Lutzomyia sandfly specific sandfly species and the area of
meal -> invade the reticuloendothelial • specific sandfly species responsible the body most affected
cells -> amastigotes for transmission vary • primarily attacks the human lymphoid
• affects the visceral tissue tissue of the skin
Epidemiology TABLE TABLE TABLE •
Clinical Symptoms • Mucocutaneous Leishmaniasis - Large • Visceral Leishmaniasis • New World Cutaneous Leishmaniasis • Old World Cutaneous Leishmaniasis
ulcers in the oral or nasal mucosa areas - Hepatosplenomegaly - single pus-containing ulcer, which is - one or more ulcers containing pus that
• cutaneous lesion may heal on its own - may resemble malaria generally self-healing generally self-heal
• Edema and secondary bacterial - incubation: 2 weeks to 18 months - pruritis - small red papule - intense itching
infections – death - skin lesions are absent - anergic - spontaneous healing of the ulcers does
- Advanced stages: kidney damage • L. pifanoi not occur
and granulomatous areas of skin - Lepromatous appearing lesions - occurs especially on the limbs and face
• L. amazonensis
• Kala-azar = “black fever”
- can progress to incurable diffuse
- Chronic cases = death in 1-2yrs TABLE
cutaneous form
- anergic px
Treatment • antimony compounds - antileishmanial • Liposomal amphotericin B • Pentavalent antimonials - sodium • sodium stibogluconate (Pentosam)
agent (Ambisome) – drug of choice stibogluconate (Pentosam) – DRUG • steroids, application of heat to the
• L. braziliensis - resistance to pentavalent • Sodium stibogluconate (Pentosam) OF CHOICE infected lesions, meglumine antimonate
antimonials (stibogluconate (Pentosam)) – also effective • Antimony combined with (Glucantime), pentamidine, and oral
• amphotericin B (Ambisome) • AIDS px – allopurinol pentoxifylline taken orally three ketoconazole
• oral antifungal drugs - fluconazole • paramomycin and miltefosine – new times a day for 30 days – SUPERIOR • Paromomycin ointment
(Diflucan), ketoconazole (Nizoral) and • Amphotericin B and liposomal
drugs
itraconazole (Sporonox)
amphotericin B (Ambisome) – also
effective
Prevention and • Public awareness through education • repellents, protective clothing, and • repellents, protective clothing, and • repellents, protective clothing, and
Control programs screening screening screening
• exercising personal protection • Prompt treatment of human • Prompt treatment of human • prompt treatment and eradication of
• prompt treatment and eradication of infections infections infected ulcers
infected ulcers • vaccine
• vaccines ongoing
Side Notes: • Can be transmitted person to person • • 16,000 cases of leishmaniasis reported in Saudi
via blood transfusions Arabia every year - arthralgias and myalgias

PARASITOLOGY RMT SOON! <3 ADM


10

CHAPTER 5: THE HEMOFLAGELLATES CONTINUATION…


Trypanosoma brucei gambiense Trypanosoma brucei rhodesiense Trypanosoma cruzi Trypanosoma rangeli
Common West African sleeping sickness, Gambian East African sleeping sickness, Rhodesian Chagas’ disease, American trypanosomiasis None known
associated disease trypanosomiasis trypanosomiasis
Laboratory • Blood, lymph node aspirations, and CSF – • trypomastigotes are blood slides • Giemsa-stained blood slides - spx of • Giemsa-stained blood – spx of
Diagnosis trypomastigote stained with Giemsa and microscopic choice choice
• Infected px – high IgM and proteins in CSF examination of CSF sediment • Lymph node biopsy Giemsa-stained • Trypomastigotes – in peripheral
• Protein and IgM studies slides & blood culture -> amastigotes blood
• CF, DAT, IIF, PCR, ELISA • Xenodiagnoses, serologic. PCR
Life Cycle Notes • tsetse fly – injects trypomastigotes via blood • Glossina morsitans and Glossina • reduviid bug vector – defecates infective • similar to T. cruzi life cycle
meal pallidipes trypomastigotes via blood meal • Rhodius prolixus – vector
• migrate in bloodstream and lymphatic system • Same life cycle with T.b. gambiense • scratching -> entry of tryps -> transforms
• multiplying by binary fission to amastigotes -> multiply & destroy the
• trypomastigotes back to the tsetse fly -> cells -> convert back into tryps ->
migration in blood -> transforms to
multiply -> salivary glands -> epimastigotes ->
amastigotes -> repeat
trypomastigotes
• can affect heart muscle, liver, and brain
• transmission - blood transfusions, sexual
intercourse, transplacental transmission,
mucous membrane
Epidemiology • tropical West Africa and Central Africa • East and Central Africa • South and Central America • South and Central America
• Glossina palpalis and Glossina tachinoides • Cattle and sheep - reservoir hosts • rarely in North America • Brazil, Venezuela, Colombia,
• first isolated in Panstrongylus megistus Panama, El Salvador, Costa Rica,
• kissing bug, conenose bug, triatomid bug Honduras, and Guatemala
- reduviid bug • monkeys, raccoons, dogs, cats,
• dogs and cats - reservoir hosts armadillos, and rodents – reservoir
hosts
Clinical Symptoms • West African (Gambian) Sleeping Sickness • East African (Rhodesian) Sleeping • Chagas’ Disease • asymptomatic
- painful chancre (ulcer) Sickness - Chagoma - erythematous nodule, 2- • benign infection
- lymphadenopathy - short incubation period 3 months
- Winterbottom’s sign - enlargement of the - myalgia and rigors - Romaña’s sign - conjunctivitis and
cervical lymph nodes - Winterbottom’s sign may or may not unilateral edema of the eyelids
- Erythematous rash, pruritis, edema be present - Megacolon, megaesophagus,
- Kerandel’s sign - delayed sensation to pain - Rapid weight loss cardiomegaly
- Somnolence - kidney damage & myocarditis – death
- most commonly seen in children
- can last as long as several years (9 to 12 months in untreated patients)
younger than 5 years
Treatment • melarsoprol, suramin, pentamidine, and • melarsoprol, suramin, pentamidine, • nifurtimox (Lampit) • Nifurtimox and benzimidazole
eflornithine and eflornithine • benznidazole, allopurinol, ketoconazole – DRUG OF CHOICE
Prevention and • destroying their breeding areas via chemical • prompt medical treatment of infected • eradication of reduviid bug nests • eradication of reduviid bug
Control treatment and clearing of brush domestic animals • construction of homes without open nests
• Proper protective clothing, repellents, and • protective clothing, screening, and design • construction of homes without
screening repellents • Educational programs open design
• prompt treatment of infected persons • clearing of brush areas and control of the • Educational programs
tsetse fly population
Side Notes: • congenital transmission • more aggressive than that of its West • cardiac and gastrointestinal distress •
African sleeping sickness • xenodiagnosis

PARASITOLOGY RMT SOON! <3 ADM


11

NOTES:
• Four morphologic forms:
o amastigote, promastigote, epimastigote, and trypomastigote
o Leishmania – amastigote
o Trypanosoma – trypomastigote
▪ Except Trypanosoma cruzi, in which amastigotes may also be found
• Blood, lymph node and ulcer aspirations, tissue biopsies, bone marrow, and cerebrospinal fluid – spx of choice

Disease or Description
Condition
Baghdad boils A common name for an infection with Leishmania tropica; it is a cutaneous form of leishmaniasis presenting with pus-containing ulcers
Bay sore A common name for a cutaneous form of infection caused by Leishmania mexicana
Chiclero ulcer A form of cutaneous leishmaniasis cased by L. mexicana; it is commonly found in Belize, Guatemala, and the Yucatan peninsula in areas where chicle sap is harvested for making chewing
gum
Dum dum fever A common name for the visceral leishmaniasis caused by Leishmania donovani
Espundia Another name for an infection resulting from Leishmania braziliensis, the principle cause of mucocutaneous disease in Central and South America, particularly in Brazil Another name for
an infection resulting from Leishmania braziliensis, the principle cause of mucocutaneous disease in Central and South America, particularly in Brazil
Forest yaws Another name for an infection with Leishmania guyanensis, the principle cause of mucocutaneous leishmaniasis in the Guianas, parts of Brazil and Venezuela; also known as pian bois
Kala-azar Another name for the most severe form of visceral leishmaniasis caused by members of the Leishmania donovani complex
Oriental sore A common reference for the cutaneous leishmaniasis caused by the infecting agents comprising the Leishmania tropic complex
Pian bois Another name for infection with L. guyanensis; also known as forest yaws
Uta A reference to mucocutaneous leishmaniasis in the Peruvian Andes

PARASITOLOGY RMT SOON! <3 ADM


12

Leishmania braziliensis Complex: Epidemiology


Subspecies Geographic Distribution Vector Reservoir Hosts
L. braziliensis Mexico to Argentina Lutzomyia and Psychodopygus sandflies Dogs and forest rodents for all
for all species comprising this complex species comprising this complex
L. panamensis Panama and Colombia
L. peruviana Peruvian Andes
L. guyanensis Guiana, Brazil, Venezuela

Leishmania donovani Complex: Epidemiology


Subspecies Geographic Distribution Vector Reservoir Hosts
L. donovani chagasi Central America, especially Mexico, West Indies, South Lutzomyia sandfly Dogs, cats, foxes
America
L. donovani donovani Parts of Africa, India, Thailand, Peoples Republic of Phlebotomus sandfly India, none; China, dogs
China, Burma, East Pakistan
L. donovani infantum Mediterranean Europe, Near East, Africa; also in Phlebotomus sandfly Dogs, foxes, jackals, porcupines
Hungary; Romania, southern region of former Soviet
Union, northern China, southern Siberia

Leishmania mexicana Complex: Epidemiology


Subspecies Geographic Distribution Vector Reservoir Hosts
L. mexicana Belize, Guatemala, Yucatan Peninsula Lutzomyia sandfly for all species Forest rodents for all species
comprising this complex comprising this complex
L. pifanoi Amazon River Basin, Brazil, Venezuela
L. amazonensis Amazon River Basin, Brazil
L. garnhami Venezuelan Andes
L. venezuelensis Venezuela

Leishmania tropica Complex: Epidemiology


Subspecies Geographic Distribution Vector Reservoir Hosts
L. aethiopica Highlands of Ethiopia, Kenya, perhaps Southern Phlebotomus sandfly for all Rock hyrax
Yemen species comprising this complex
L. major Former Soviet Union, Iran, Israel, Jordan, parts of Gerbils, other rodents
Africa, Syria (esp. in rural areas)
L. tropica Mediterranean, parts of the former Soviet Union, Possibly dogs
Afghanistan, India, Kenya, Middle East (especially in
urban areas)

PARASITOLOGY RMT SOON! <3 ADM


13
CHAPTER 6: PLASMODIUM AND BABESIA
Plasmodium vivax Plasmodium ovale Plasmodium malariae Plasmodium falciparum
Benign tertian malaria, vivax malaria Benign tertian malaria, ovale malaria Quartan malaria, malarial malaria Black water fever, malignant tertian malaria,
Common
aestivoautumnal malaria, subtertian malaria,
associated disease
falciparum malaria
Laboratory • All morphologic stages may be seen on thick • blood film preparations • ring stage not usually seen • Peripheral blood smears
Diagnosis and thin peripheral blood films • all stages of development may be • developing trophozoite and the • Mild to moderate infection - ring forms and
• best time to observe - halfway between seen immature and mature schizonts - gametocyte
paroxysms frequently encountered • Severe infection - trophozoites and schizonts
• gametocytes – seen but not readily • Hypnozoites are not produced
distinguishable • relapses are not known
• thick and thin Giemsa-stained • recrudescence may occur - fatal
peripheral blood films
Life Cycle Notes • invade young RBCs • infects young RBCs • infects mature RBCs • infect RBCs at any age
• distortion of the RBCs • ability to adapt • well-established cell walls - not • occur in warmer months of late summer and
conducive to expansion early autumn
Epidemiology • worldwide - tropics and subtropics • in tropical Africa - surpassed P. • subtropic and temperate regions • tropical and subtropical regions
• temperate regions vivax frequency • occur less frequently
Clinical Symptoms • Benign Tertian Malaria • Benign Tertian Malaria and Ovale • Quartan or Malarial Malaria • Black Water Fever and Malignant Tertian
- 10- to 17-day incubation period Malaria - incubation period of 18 to 40 days Malaria
- Mimic symptoms of flu - paroxysm cycle (every 48 hours), - flulike symptoms - Incubation period of 7 to 10 days
and relapses - paroxysms every 72 hours
- Dormant hypnozoites - relapses months - Mimic malignant infections symptoms
- infections last approximately 1yr - no relapses – no dormant hypnozoites
to years after initial infection - Kidney involvement – hemoglobinuria
- relapse infections recover - repeated attacks for 20 yrs or more
spontaneously - Coma or death
Treatment • quinine, quinidine, chloroquine, • same as P. vivax • same as P. vivax • same as P. vivax
amodiaquine, primaquine, pyrimethamine,
sulfadoxine, dapsone, mefloquine,
tetracycline, doxycycline, halofantrine,
atovaquone, proguanil, ginghaosu,
artemisinin, artemether, artesunate,
pyronaridine, Fenozan B07, trioxanes, nonane
endoperoxides, azithromycin, and WRZ38605
Prevention and • netting, screening, protective clothing, and • same as P. vivax • same as P. vivax • prophylactic therapy
Control repellents • proper personal protection
• prophylactic treatment • chemically destroying breeding areas
• avoidance of sharing intravenous needles • screening of donor blood units
• no vaccine yet • avoidance of sharing IV needles
• vaccine - RTS,S/AS01: GlaxoSmithKline
Biologicals’
Side Notes: • recombinant DNA probes and ribosomal RNA • Schüffner’s dots • Ziemann’s dots - fine dustlike dots • Maurer’s dots - dark-staining, irregular to
probes – diagnosis of malaria comma-shaped
• Schüffner’s dots - eosinophilic stippling, tiny
granules in the cytoplasm

PARASITOLOGY RMT SOON! <3 ADM


14
CHAPTER 6: PLASMODIUM AND BABESIA CONTINUATION…
Plasmodium knowlesi Babesia microti (Theileria microti) Babesia divergens
Common associated disease Presently, no common name exists Presently, no common name exists
Laboratory Diagnosis • DNA extraction and nested-PCR examination • Giemsa-stained peripheral blood films – spx of choice
• Wright’s stain may also be used
• studied under oil immersion
• serologic tests and PCR techniques
Life Cycle Notes • morphologically resembles P. malariae • genus Ixodes – tick vector
• uninfected host must be in contact with the tick’s saliva for 12 hours or longer to transmit
infection
• sporogony—spore and sporozoite production via sexual reproduction
Epidemiology • Malaysia and other parts of Southeast Asia • southern New England, such as Nantucket, • European countries, particularly those in the
Martha’s Vineyard, Shelter Island, Long Island, former Yugoslavia, Russia, Ireland, and Scotland
and Connecticut, New Jersey, Wisconsin, • Vector - Ixodes Ricinus
Missouri, Georgia, North Carolina, and Mexico • principal reservoir hosts - cattle and rabbits
• Ixodes dammini - vector
• principal reservoir host - Peromyscus leucopus,
white-footed mouse
Clinical Symptoms • respiratory distress, acute renal or multi-organ • Px was exposed 1 to 4 weeks prior to the onset of symptoms
failure, to shock • self-limiting
• prodrome-like symptoms
• no periodicity
• Elevated bilirubin and transaminase
• Worse in splenectomized and immunocompromised
• coinfection with Lyme disease and/ or human granulocytic ehrlichiosis
Treatment • quinine, chloroquine or artemetherlumefantrine • clindamycin and quinine or atovaquone and azithromycin
– px w/o complications • Diminazene and pentamidine
• Inraveous quinine, artesunate or a combination • pyrimethamine and quinine
of chloroquine-primiquine – for severe infections • Patient age, immune status, G6PD status – must be checked
• Artemisinin – both mild & severe
Prevention and Control • • A • A
Side Notes: • Parasite of Old World Monkeys • transfusion-transmissible disease
• transmitted congenitally
• sharing of intravenous drug needles

PARASITOLOGY RMT SOON! <3 ADM


15
NOTES: • 3 paths of merozoites:
PLASMODIUM o Rupture & infect new RBCs
• General Cornwallis surrendered after his soldiers were exposed to malaria o develop into microgametocytes and macrogametocytes
• North America was declared malaria free in 1970 o destroyed by immune system of healthy individuals
• Ring Forms (Early Trophozoites) • hypnozoites - dormant Plasmodium-infected liver cells; sleeping forms
- Ringlike • recrudescence - relapse infection
- invasion into a previously healthy RBC • gametocytes - male (micro) and female (macro) sex cells
- vacuole - space inside the ring • zygote (ookinete) - fertilized cell
- blue cytoplasmic circle • oocyst – encysted zygote
- nucleus - red chromatin dot • Transfusion malaria
• Developing Trophozoites • Giemsa-stained peripheral blood films – spx of choice, recommended stain
- Brown pigment • Thick blood smears - screening slides
- More RBC space invaded • thin blood smears – differentiating species
• Immature Schizonts • mixed infections - P. vivax and P. falciparum
- Brown pigment • paroxysms - fever and chills (rigor) 10 to 15 minutes or longer
- unorganized, active chromatin • greatest number of parasites in between paroxysms
• Mature Schizonts • blood collection every 6 to 12 hours for up to 48 hours – malaria-free
- MEROZOITES - fully developed stage of the asexual sporozoan • rigor - chills
- cytoplasmic material is not visible - except P. vivax
• Microgametocytes BABESIA
- Round – except P. falciparum (crescent-shaped) • Texas cattle fever or red water fever
- chromatin mass - pink to purple
• Merozoites undergo binary fission
• Macrogametocytes
• has sexual and asexual phase
- round to oval - except P. falciparum (crescent-shaped)
• sexual phase – w/in the vector (tick)
• schizogony - asexual multiplication
• asexual phase – w/in host (mice, deer, cattle, dogs, humans)
• exoerythrocytic cycle - reproduction outside of RBC
Occurrence of Cyclic Paroxysms in Common Plasmodium Species
Plasmodium Species Timing of Cyclic Paroxysms
P. vivax Every 48 hr
P. ovale Every 48 hr
P. malariae Every 72 hr
P. falciparum Every 36-48 hr

PARASITOLOGY RMT SOON! <3 ADM


16

PARASITOLOGY RMT SOON! <3 ADM


17
CHAPTER 7 - MISCELLANEOUS PROTOZOA
Balantidium coli Isospora belli Sarcocystis species Cryptosporidium parvum Blastocystis hominis Cyclospora cayetanensis

Common
Blastocystis hominis Cyclospora cayetanensis
associated Balantidiasis Isosporiasis Sarcocystis infection Cryptosporidosis
infection infection
disease
Laboratory • Stool – presence of • fresh feces and duodenal • Stool -> fully developed • Stool – oocysts • Stool • Stool – without formalin
Diagnosis trophs and cysts contents – oocysts oocysts - iodine or modified acid- • Iodine wet • sporulate best at room
• Sigmoidoscopy • Enterotest – oocysts • only the sporocysts are fast stain – stain of preparations temperature
material – • Intestinal biopsy – intracellular visible on examination choice - Cytoplasm- light • 5% potassium
sigmoidorectal morphologic stages, oocyst - singly or in pairs that - formalin fixed smears yellow dichromate – makes
infection • Stool – may be free from appear to be with Giemsa-stain - vacuole – sporocysts visible
• wet preparations and parasite cemented together - Enterotest, ELISA, and transparent • Flotation methods using
the permanent stain • oocysts may be visible in direct • Routine histologic indirect • saline – lyses B. hominis phase contrast or bright
• multiple samples is wet preparations methods – identify immunofluorescence -> false-negative result field microscopy
needed - appear transparent in Sarcocystis cyst in human - Sheather’s sugar • Modified acid-fast stain
• macronucleus is always saline wet prep muscle samples flotation • Oocysts autofluoresce –
visible in the stained - iodine wet prep – standard - Sheather’s sugar UV light microscopy
cyst and troph processing of spx flotation – phase
- dec. microscope light level contrast microscopy
& proper contrast - Intestinal biopsy -
• Sheather’s sugar flotation merozoites and
• Acid-fast stain – rec. gametocytes
permanent stain for
confirmatory identification
Life Cycle • similar to E. histolytica • no intermediate hosts • morphology of the oocysts Merozoite, • reproduces by • ingestion of an oocyst
Notes • ingestion of infective • humans – definitive hosts resembles that of Isospora microgametocytes, and sporulation or binary • two sporocysts enclose
cysts • ingestion of infective • asexual reproduction in the macrogametocytes: 4-8 fission two sporozoites
• multiplication does not mature/sporulated oocysts intermediate host Size: 2 to 4 µm • sexual and asexual • asexual reproduction ->
occur in the cyst phase • excystation of oocyst -> • transmission: • This morphologic form is reproduction merozoites
• multiplication of trophs sporozoites - ingestion of uncooked not routinely seen in • exhibits pseudopod • sexual reproduction ->
– transverse binary • schizogony -> merozoites pig or cattle meat patient samples, but is extension and microgametocyte
fission – 2 young (intestinal mucosa) infected with required to complete the retraction production
trophs emerge • macrogametocytes and Sarcocystis sarcocysts life cycle of • detailed discussion of • Infected humans pass
• trophs do not survive microgametocytes (intestinal -> human definitive Cryptosporidium the B. hominis life cycle immature oocysts in the
outside environment mucosa) -> oocysts -> excreted host • excystation in the upper has not been widely stool
• encystation – lumen in stool - humans accidentally gastrointestinal tract -> presented • No animal reservoir
• excystation – cecal & • oral-anal sexual contact swallow oocysts from sporozoites exists.
terminal portion of stool sources of • asexual and sexual
ileum animals other than replication
cattle or pigs. -> • two forms of oocysts:
human intermediate - thin-shell:
host autoinfections
- thick shell: usually
remains intact and is

PARASITOLOGY RMT SOON! <3 ADM


18
passed out of the
body
Epidemiology • Human infection is very • Africa, Southeast Asia, and • frequency of infections is • 20 species known to exist • epidemics in • United States and Canada
low Central America, South relatively low • water or food subtropical countries • Lima, Peru, Nepal and
• Epidemic – psychiatric America, particularly in Chile contaminated with • ingestion of fecally parts of Asia
facilities in US • Increase frequency in AIDS px infected feces contaminated food or • Chicago, 1990 – source of
• Oral-fecal & person-to- • Opportunistic infection • person-to-person water minioutbreak
person transmission • Contaminated lettuce
• Pig – reservoir host • Immunocompromised and imported fresh fruit
persons at risk (raspberries) – source of
• immunocompetent infection
children in tropical areas,
children in day care
centers, animal handlers,
and those who travel
abroad.
Clinical • Asymptomatic – • Asymptomatic -> self-limited • Sarcocystis Infection: • self-limiting diarrhea – • pathogenicity of B. • similar to those seen in
Symptoms similar to E. histolytica • Isosporiasis – mild - compromised lasts 2 weeks hominis is not totally cases of cryptosporidiosis
• Balantidiasis gastrointestinal discomfort to patients • if fluid loss is great – fatal clear • difference – longer
- Mild colitis and severe dysentery - fever, severe in children • B. hominis in absence diarrhea in C.
diarrhea to full-blown • Eosinophilia diarrhea, weight loss, • Malabsorption of another intestinal cayetanensis
clinical balantidiasis • Charcot-Leyden crystals – and abdominal pain • debilitating condition pathogen - diarrhea,
(resemble amebic stool samples • muscle tenderness – • infection rates in AIDS px: vomiting, nausea, and
dysentery) • malabsorption syndrome invasion of striated muscle - 3 to 20% in the United fever, abdominal pain
- Abscesses and ulcer • Infected patients may shed States and cramping
with secondary oocysts in their stools for as - 50 to 60% in Africa and • B. hominis in addition
bacterial infection long as 120 days Haiti to another pathogenic
- Chronic: tender colon, organism – severe
anemia, cachexia, symptoms
occasional diarrhea
- B. coli can infect liver,
lungs, pleura,
mesenteric nodes, and
urogenital tract
• Extraintestinal
infections is rare
Treatment • Asymptomatic – good • consuming a bland diet and • humans definitive host: • Spiramycin – helpful but • Iodoquinol or -
chance of recovery obtaining plenty of rest - treatment similar to I. still in the experimental metronidazole
• Oxytetracycline • severe infections -> belli stage
(Terramycin), chemotherapy - trimethoprim +
iodoquinol, - combination of sulfamethoxazole
metronidazole trimethoprim and - pyrimethamine +
sulfamethoxazole or sulfadiazine
pyrimethamine and • humans intermediate
sulfadiazine host:

PARASITOLOGY RMT SOON! <3 ADM


19
- no known specific
chemotherapy to treat
Sarcocystis
Prevention - Personal hygiene and • similar to those of E. • humans definitive host: • Proper treatment of water • Proper treatment of • properly treating water
and Control proper sanitary histolytica - adequate cooking of beef supplies fecal material • only using treated water
conditions • proper personal hygiene and pork • PPE • hand washing
• adequate sanitation practices • humans intermediate • hand washing • proper handling of
• avoidance of unprotected sex, host: • disinfecting potentially food and water
particularly among homosexual - proper care and disposal infected equipment with
men. of animal stool full-strength commercial
bleach or 5% to 10%
household ammonia
Side Notes: • Trophs – “sac” in shape • S. hovihominis or S. • first associated with • was given its current • may not be recovered
• Balantidium = “little hominis – Cattle poultry and cattle name in 1912 by Emile using standard or
bag” • S. suihominis – pigs • neonatal diarrhea in calves Brumpt traditional specimen
• 63% to 91% of pigs • S. lindemanni - umbrella and lambs • Initially considered as processing techniques
harbor B. coli term for those organisms • 1976 – first reported an algae, then a
that may potentially • 1987 – Carroll County, harmless intestinal
parasitize humans. Georgia yeast, then a protozoan
• oval transparent organism • 2011 – Indiana parasite
• layering and flotation of • Blastocystis is not
the sample over a fungal or protozoan
hypertonic sodium • Stramenopiles, a major
chloride solution – line of eukaryotes.
separates Cryptosporidium
oocysts from fecal debris

PARASITOLOGY RMT SOON! <3 ADM


20
CONTINUATION
Microsporidia Toxoplasma gondii Pneumocystis jiroveci
Common Microsporidia infection, microsporidial infection. Toxoplasmosis, congenital toxoplasmosis, cerebral toxoplasmosis
Pneumocystosis, atypical interstitial plasma cell
associated
pneumonia
disease
Laboratory • Serologic tests, cell culture, staining, • Serologic test - primary means of diagnosis • Giemsa and iron hematoxylin stains –
Diagnosis - trichrome or acid-fast stain (ACF) • Determination of IgM in congenital infections – double-sandwich ELISA diagnosis
- ACF or PAS – gram positive • IgG and IgM – IFA test - Gomori’s methenamine silver nitrate stain
- Giemsa-stain - biopsy material and fecal • IgG – ELISA and IHA • Serologic techniques - not yet for clinical
concentrate diagnosis
• Transmission Electron Microscopy – speciation • monoclonal immunofluorescent stain -
identification.
• Sputum – immunocompromised
• bronchoalveolar lavage, tracheal aspirate,
bronchial brushings, and lung tissue
Life Cycle • May have intermediate host • Cat - definitive host • unknown
Notes • infective spores inject sporoplasm into a host cell • ingestion of T. gondii cysts from mice or rats (intermediate host) • takes up residence in the alveolar spaces in
- bradyzoites are released and transforms into tachyzoites
• dispersed into the outside environment by feces, lung tissue
• sexual and asexual – cat’s gut
urine, or by death of the host • mature cysts -> trophozoites -> precysts and
• sexual -> oocysts -> shed in stool
cysts
• mature oocyts -> sporozoites -> tachyzoites (intestinal epithelium of rodent)
• tachyzoites migrate into the brain or muscle of rodents -> bradyzoites • also infects the spleen, lung, lymph nodes, and
• human infection is accidental bone marrow.
- hand-to-mouth transmission contact with cat
- ingestion of undercooked cattle, pig, or sheep meat -> cat’s feces
- transplacental infection
- blood transfusion (rare)
Epidemiology • AIDS patients from Haiti, Zambia, Uganda, the • AIDS px • United States, Asia, and Europe
United Kingdom, the United States, and the • occur in 15% to 20% of the population in the United States • transfer of pulmonary droplets through direct
Netherlands • 4000 infants - transplacentally acquired - US each year person-to-person contact
• oocysts survived up to 18 months • immunosuppressed patients, AIDS px
• malnourished children w/ malignancy
Clinical • enteritis, keratoconjunctivitis, and myositis • Asymptomatic • Pneumocystosis: Atypical Interstitial Plasma
Symptoms • peritonitis and hepatitis - rare • Only cause disease in humans if: Cell Pneumonia.
- virulent strain entered the body - nonproductive cough, fever, rapid
- susceptible host (AIDS)
respirations, and cyanosis.
- site in human body where tissue destruction can occur
- leading cause of death in AIDS patients
• Toxoplasmosis
- mimic infectious mononucleosis - AIDS px infected w/ P. jiroveci also suffer
- maculopapular rash from Kaposi’s sarcoma
- encephalomyelitis - infiltrate on chest x-ray
- myocarditis & hepatitis - low PO2
- Retinochoroiditis (rare) - poor prognosis
• Congenital Toxoplasmosis
- severe and often fatal
- 1-5 of every 1000 pregnancies
- Degree of severity: antibody protection from mother & age of fetus at time of
infection

PARASITOLOGY RMT SOON! <3 ADM


21
- Retinochoroiditis
- hydrocephaly, microcephaly, intracerebral calcification, chorioretinitis,
convulsions, and psychomotor disturbances
- mental retardation, severe visual impairment, or blindness
- 5% to 15% of infants will die
- 10% to 13% of infants - moderate to severe handicaps.
- 8% to 10% - Severe eye and brain damage
- 58% to 72% - asymptomatic at birth
• Hodgkin’s lymphoma – opportunistic toxoplasmosis
• do not spread into other organs of the body but rather stay confined within the CNS
• rise in spinal fluid IgG – diagnostic
• most infected px – no serum IgM levels
• lack of serum IgM + lack of change in serum IgG -> chronic latent infection
Treatment • Albendazole – recommended for E. bieneusi • trisulfapyrimidines + pyrimethamine (Daraprim) • Trimethoprim-sulfamethoxazole (Bactrim) –
- oral fumagillin - alternative • pyrimethamine – not for pregnant women first line treatment
• Albendazole plus fumagillin eye drops – for • spiramycin – alternative • Pentamidine isethionate and cotrimoxazole -
Nosema infection • Folinic acid (leucovorin) – AIDS px alternative treatments
• Atovaquone – encephalitis in AIDS px
Prevention - - avoidance of contact with cat feces - difficult to implement
and Control - hand washing - PPE
- avoidance of ingesting contaminated meat -
- keeping cats away from potentially infective rodents
Side Notes: • protozoal disease BUT Fungi • 1908 - African rodent Ctenodactylus gondii • formerly called Pneumocystis carinii
• Three of the five genera – cause human disease • venous blood from AIDS patients • fungus
(AIDS px) • amniotic fluid from pregnant women •
• Enterocytozoon bieneusi - most well-known • Immunocapture assay – diagnosis in pregnant women
• Encephulitozoon and Pleistophora – AIDS px
• corneal infections & Nosema (fatal in IC infant)
• emerging water-borne pathogen
• EPA Candidate Contaminate List
• both the infective and diagnostic stages are
spores

MISCELLANEOUS PROTOZOA
• All are unicellular
CLASSIFICATION:
1. Ciliates - Balantidium coli
2. Plasmodium and Babesis spp. – Coccidia
3. Blastocystis hominis
4. Pneumocystis jiroveci – (Pneumocystis carinii)

CAT – Toxoplasmosis
CATTLE/COW – Sarcosystis, Cryptosporidosis, Toxoplasmosis
PIG – Sarcosystis, Toxoplasmosis, Balantidiasis

PARASITOLOGY RMT SOON! <3 ADM


22
CHAPTER 8: THE NEMATODES
Enterobius vermicularis Trichuris trichiura Ascaris lumbricoides Hookworms
Necator americanus (New World
hookworm)
Ancylostoma duodenale (Old World
hookworm)
Common associated Enterobiasis, pinworm infection Trichuriasis, whipworm infection Ascariasis, roundworm infection Hookworm infection, ancylostomiasis,
disease necatoriasis
Laboratory Diagnosis • Spx of choice - cellophane tape preparation • Spx of choice – stool • Stool -> eggs • Stool -> eggs
• Eggs – primary form seen • Infective eggs • ELISA • Larvae may mature and hatch from eggs
• Adult females – perianal region • Zinc sulfate flotation method in stool that has been allowed to sit at
• Multiple sample -> confirm the presence & • Distorted eggs -> variety of room temperature, without fixative
determine if px is free from infection unusual shapes added.
Life Cycle Notes • Humans – ONLY HOST • Adult – colon • Ingestion of infected eggs w/ larvae -> • third-stage filariform larvae – skin
• Ingestion of infected EGGS • After 1 month outside -> eggs Small intestine -> larvae -> liver -> penetration (FEET)
- Self-limiting embryonate -> become infective - bloodstream -> lung -> larvae burrow in - migrates to lymphatics and blood
• Small intestine -> hatch & release larvae > begins new life cycle alveolar capillaries -> migration in system
• Colon -> adult worm bronchioles -> pharynx -> returned to - lungs -> penetrate capillaries,
• Copulation (mating) – gravid (pregnant) intestine -> larva matures -> adult worms enter alveoli -> bronchioles ->
worm in the intestine -> multiply up to 250,000 pharynx (by coughing) ->
- 15,000 eggs in perianal region undeveloped eggs/day -> feces -> swallowed -> intestine
• Eggs may become airborne embryonated in soil (maturation occurs)
• Ideal surrounding for thriving infective • Eggs can still survive in 10& formalin • 10,000 to 20,000 eggs/day
eggs: fixative • Passed on to the feces
- Moderate temp • EMBRYONATED EGGS -> infective stage • Eggs -> warm, moist soil -> become
- High humidity first-stage rhabditiform -> molt twice ->
• Retroinfection – pinworm eggs that third-stage filariform larvae
migrate back into the body
• Autoreinfection – infected individuals
reinfect themselves
Epidemiology • most common helminth known to infect • 3rd most common helminth • Most common intestinal helminth -> 1 • almost 25% of the world’s population is
humans in US • Warm climates billion affecting worldwide infected with hookworm
• Children – at great risk • Children & psychiatric facilities • Warm climates & poor sanitation • Warm climates & poor sanitation
• Hand-to-mouth contamination • Trichuris & Ascaris – can exist in • Children -> at risk • Mixed infections with Trichuris and
• E. vermicularis – transmission of D. fragilis both infection • Source of contamination -> children’s toys Ascaris are possible
to soil • N. americanus is primarily found in
North and South America
• A. duodenale may be found in Europe,
China, Africa, South America, and the
Caribbean.
Clinical Symptoms • Asymptomatic • Asymptomatic • Asymptomatic • Asymptomatic
• Enterobiasis: • Trichuriasis: - 5-10 worms - diet rich in iron, protein, and other
- Intense itching & inflammation in - 500-5000 worms • Ascariasis vitamins maintains asymptomatic
anal/vagina - Children w/o good eating habits -> state
- Intestinal irritation protein malnutrition • Ancylostomiasis, Necatoriasis
PARASITOLOGY RMT SOON! <3 ADM
23
- Minute ulcers - Children infected -> - Secondary bacterial infections - intense allergic itching (GROUND
resemble ulcerative ITCH)
colitis - chronic infections -> most common
- Adults infected -> form seen – <500 eggs/g of feces
mimics inflammatory - acute infections - >5000 eggs/g of
bowel disease feces
- 200 worms -> - Secondary bacterial infections
dysentery, severe
anemia, growth
retardation
- RECTAL PROLAPSE &
PERISTALSIS
Treatment • albendazole, mebendazole, or pyrantel • albendazole or mebendazole • albendazole & mebendazole • mebendazole and pyrantel pamoate
pamoate • Asymptomatic - iron replacement
and/or other dietary therapy
Prevention and • personal hygiene • proper sanitation • avoidance of using human feces as • proper sanitation
Control • handwashing • avoidance defecating directly in fertilizer • fecal disposal
• ointment soil • Proper sanitation • prompt and thorough treatment of
• avoid scratching • avoiding feces as fertilizer • Personal hygiene infected persons
• educating children • PPE
• personal hygiene
Side Notes: • unfertilized eggs: • two primary differences
- oblong - geographic distribution
- thin shell - adult worms morphology
- corticated • primordium - precursor structure to a
• fertilized eggs: reproductive system consisting of a
- rounded clump of cells in an ovoid formation
- thick shell (CHITIN) • Filariform Larvae – INFECTIVE, non-
- decorticated eggs may be present -> feeding > emerges from rhabditiform
lacking outer mamillated albuminous - slender larva has a shorter
coating esophagus than that of
Strongyloides stercoralis
- distinct pointed tail
• Adults
- Rarely seen
• indoor plumbing – decrease hookworm
infections
• alternative hookworm infection
diagnosis - reverse enzyme
immunoassay for IgE
• Ancylostoma braziliense (dogs and
cats) and Ancylostoma caninum (dogs)
– accidental in humans
- Cutaneous larva migrans
- Treatment: thiabendazole

PARASITOLOGY RMT SOON! <3 ADM


24

CHAPTER 8 – THE NEMATODES CONTINUATION….


Strongyloides stercoralis (Threadworm) Trichinella spiralis (Trichina worm) Dracunculus medinensis (Guinea worm)
Common Strongyloidiasis, threadworm infection Trichinosis, trichinellosis Dracunculosis, dracunculiasis, guinea worm infection
associated disease
Laboratory • Stool -> eggs • Method of choice -> examination of the affected skeletal • Adult D. medinensis worms - infected ulcers
Diagnosis • Stool concentration with zinc sulfate muscle -> encysted larvae • immersing in cool water - reveals the first-stage larvae
• rhabditiform larvae -> fresh stool samples and • eosinophilia and leukocytosis (rhabditiform)
duodenal aspirates • increased LDH, aldolase, and creatinine phosphokinase
• Enterotest • No known test is completely 100% accurate

Life Cycle Notes • three possible routes: direct, indirect, autoinfection • normal host – animal • Ingestion of drinking contaminated water (copepods -
• rhabditiform larvae passed on to feces • human – accidental host freshwater fleas)
• remaining phases of the threadworm life cycle • no egg stage in this life cycle • Copepods – contains D. medinensis third-stage larvae
basically mimic those of the hookworm life cycle • granuloma forms -> become calcified - Consumes first-stage larvae as its intermediate
• Indirect life cycle - threadworm rhabditiform larvae -> • doesn’t complete life cycle host
outside environment -> free-living adults • Adult worms – intestinal wall
(nonparasitic) -> filariform state -> new indirect cycle • Gravid female worms – subcutaneous tissue -> lay first
or begin direct cycle stage larvae
• Autoinfection – rhabditiform larvae develop into the - Adult females may escape from the body or
filariform stage inside the intestine of the human host migrate back into deeper tissues
-> enter lymphatic system or bloodstream -> new - Adult males – not known
cycle of infection

Epidemiology • tropical and subtropical regions of the world • found worldwide except tropics • Africa, India, Asia, Pakistan, and the Middle East
• resistant to colder regions • Copepods reside in fresh water called step wells
• feeding of contaminated pork scraps to hogs • reservoir host - dogs

Clinical Symptoms • Asymptomatic – light infection • great imitator • allergic reactions


• Strongyloidiasis: • may result in death • Secondary bacterial infections – can cause disability or
- diarrhea and abdominal pain • light infection - diarrhea and slight fever even death
- Urticaria w/ eosinophilia • heavy infection - vomiting, nausea, abdominal pain, • nodule formation
diarrhea, headache, eosinophilia, pain in the pleural area,
fever, blurred vision, edema, and cough

PARASITOLOGY RMT SOON! <3 ADM


25
Treatment • Ivermectin • No medications – if not life-threatening • no specific dracunculiasis medicines available
• Albendazole – alternative • Prednisone – if life-threatening • total worm removal – successful treatment
• Thiabendazole – may be given but effectiveness is • 5 steps:
questionable - 1: place affected part in cool water
• Steroids - 2: adult worm breaks through the blister
- 3: clean the resulting wound
- 4: manual extraction
- 5: topical antibiotics
Prevention and • proper handling and disposal of fecal material • Thorough cooking of meats • use of properly treated water
Control • protection of the skin from contaminated soil • below-zero temperatures storage of meat for 20days – • prohibiting the practice of drinking and bathing in the
decrease the viability of T. spiralis same water
• avoidance of feeding pork scraps to hogs • standing water not to be ingested
• highly unlikely of total eradication of guinea worm

Side Notes: • one of the largest adult nematodes


• dates back to biblical times
• fiery serpents
• plague - Israelites who lived by the Red Sea

NEMATODA: • Developing larvae • pinworms – ingestion of infected eggs


• Multicellular, round in cross section o Inside fertilized egg • hookworms – burrow through the skin
• Phylum Nemathelminthes o Long & slender • adult worms – intestine
• 2 groups: • When larva in the eggs is outside the body: • Trichinella and Dracunculus - tissue is involved
o Intestinal spp o Requires warm moist soil • 3 factors in severity of nematode infection
o Intestinal-tissue spp o 2-4 weeks to develop o number of worms present
• 3 basic morphologic forms: • Adult female worms are usually larger than the o length of time the infection persists
o Eggs (female sex cells) adult males o overall health of the host
o Larvae (juvenile worms) o complete digestive and reproductive • infections can last up to 12 months or longer
o Adult worms systems o reinfections and/or autoinfections

PARASITOLOGY RMT SOON! <3 ADM


26

PARASITOLOGY RMT SOON! <3 ADM


27

PARASITOLOGY RMT SOON! <3 ADM


28
CHAPTER 9: THE FILARIAE
Wuchereria bancrofti (Bancroft’s Brugia malayi Loa loa Onchocerca volvulus Mansonella ozzardi Mansonella perstans
filaria) (Malayan filaria) Eye worm (African) (Blinding filaria) (New World filaria) (Perstans filaria)
Common Bancroft’s filariasis or Malayan filariasis or elephantiasis Loiasis River blindness, nonpathogen nonpathogen
associated elephantiasis onchocerciasis
disease
Laboratory • fresh Giemsa-stained blood -> • stained blood films – best method of • Giemsa-stained blood • Multiple Giemsa-stained • peripheral blood -> • Blood – specimen of
Diagnosis microfilariae diagnosis – spx of choice for slides of tissue biopsies microfilariae choice
• more sensitive method - • nocturnal periodicity recovery of (skin snips) - Recovery of • nonperiodic • non-periodic
filtering heparinized blood • Subperiodic organisms may appear microfilariae microfilariae
using nuclepore filter • Knott technique • Sample collection: • Adult worms - infected
• Knott technique between 10:15 AM nodules
• Light infections - 1 mL of blood and 2:15 PM • Eye – examined using slit
in 10 mL of a 2% solution of • diurnal lamp
formalin • Eosinophilia, Calabar • eosinophilia and ocular
• optimal sample is collected at or transcient discomfort
night (nocturnal periodicity) subcutaneous • serologic and PCR
• Peak hours: 9:00 p.m. and swellings – helps in
4:00 a.m diagnosis
• antigen and antibody •
detection and PCR assays
Life Cycle • Culex, Aedes, and Anopheles • Aedes, Anopheles or Mansonia • bite of an infected • blackfly genus Simulium • injection of infective • Life cycle similar to
Notes spp. – intermediate host & • Co-infection with W. bancrofti I – Chrysops fly • encapsulate in larvae to the human that of M. ozzardi
vectors theoretically possible. • residence and multiply subcutaneous fibrous definitive host • Culicoides sucking
• human host – adult worms • life cycle similar to that of W. throughout the tumors • Culicoides sucking midge - only known
reside in lymphatics and lay bancrofti subcutaneous tissues • rarely seen in the midge or Simulium vector
microfilariae • difficult to diagnose peripheral blood blackfly - vectors • settle in the eye
• Microfilariae - • humans - primary
blood, capillaries definitive hosts
and intravascular
spaces of the skin
• Adults - body
cavities, visceral fat,
and mesenteries
Epidemiology • subtropical and tropical areas • Philippines, Indonesia, Sri Lanka, New • Africa - rainforest belt • equatorial Africa and • exclusively in the • Africa, selected
of the world Guinea, Vietnam, Thailand, and region Central America Western areas in the
• Africa, the Nile Delta, India, specific regions of Japan, Korea, and • East Africa, Zaire, Angola, Hemisphere Caribbean Islands,
Pakistan, Thailand, the Arabian China. parts of Mexico, • known to exist in Panama, and
sea coast, the Philippines, • humans - primary definite host Colombia, Brazil, and North America, northern South
Japan, Korea, and China in the • infect felines and monkeys portions of Venezuela Central and South America
Eastern Hemisphere and in • breeds in running water Americas, West
Haiti, the Dominican Republic, Indies and
Costa Rica, and coastal Brazil in Caribbean
the Western Hemisphere

PARASITOLOGY RMT SOON! <3 ADM


29
Clinical • Asymptomatic • Asymptomatic • Loiasis • Onchocerciasis: River • asymptomatic • damage to affected
Symptoms - Adult px who were exposed • Fevers may take months to years to • Pruritus Blindness infections are tissue is minimal
when they were children develop after the initial infection • localized pain • chronic and nonfatal common • minor allergic
- Microfilariae recovered in • granulomatous lesions • Calabar swellings • development of infected • urticaria, reactions, or no
blood samples • lymphatics, chills, lymphadenopathy, • adult worms – only nodules lymphadenitis, skin symptoms at all
- Eosinophilia lymphangitis, and eosinophilia noticeable when • severe allergic reactions itching, and • Calabar swellings
- enlarged lymph nodes migrating to to microfilariae arthralgias • may be responsible
- self-limiting conjunctiva or bridge • may lead to blindness for joint and bone
• Symptomatic Bancroftian of the nose • Scratching leads to pain
Filariasis secondary bacterial
- granulomatous lesions, infections.
lymphangitis, and
lymphadenopathy
- Bacterial infections with
Streptococcus
- Elephantiasis (swelling of the
lower extremities)
- dead adult worms calcified
and form abscesses
Treatment • diethylcarbamazine (DEC) and • Similar to W. bancrofti but • Surgical removal – • ivermectin • Asymptomatic • Asymptomatic
ivermectin (Stromectol) when diethylcarbamazine (DEC) is the treatment of choice • no known effective infections are not infections are often
used in combination with most useful • diethylcarbamazine medication without toxic typically treated not treated.
albendazole • anti-inflammatory drugs (DEC) - medication of effects • Ivermectin – for px • Diethylcarbamazine
• Surgical removal of excess choice • adult worms can live up who require (DEC) – treatment of
tissue – for scrotum to 15 years. treatment choice
• Unna’s paste boots, elastic • Diethylcarbamazine • Mebendazole –
bandages and simple elevation (DEC) - INEFFECTIVE alternative
- reduce the size of an infected • Ivermectin –
enlarged limb INEFFECTIVE
Prevention • personal protection • Similar to W. bancrofti • Personal protection • personal protection • no known control • use of insecticides
and Control • destroying breeding areas of • destroying the vector • use of insecticides programs in place at • Personal protection
the mosquitoes breeding areas this time
• insecticides • use of prophylactic • vectors are so small
• educating the inhabitants of DEC – not deterred by
endemic areas nets or screening
• Mosquito netting and insect equipment
repellants
Side Notes: • date back as far as the second • may also be caused by Brugia timori
millennium BC (1964 on the island of Timor)
- microfilariae: 310 µm
• early explorers of the 17th and
- sheath: difficult to observe using
18th centuries – learned about
Giemsa stain
bancroftian filariasias when - does not bulge around the two nuclei
they visited Polynesia • tropical eosinophilia (occult filariasis) –
people who reside where both B. malayi
and W. bancrofti are endemic
- pulmonary and asthmatic symptoms
- no microfilariae in blood

PARASITOLOGY RMT SOON! <3 ADM


30

NOTES:
• filariae or filarial nematodes
• adult worms live in tissue or the lymphatic system
o produce microfilariae
▪ exhibit periodicity (helpful in speciating the microfilariae forms)
• nocturnal (occurring at night), diurnal (occurring during the day), or subperiodic (timing of occurrences not clear-cut)
o vectors – biting insects (mosquitoes)
o Asia, Africa, South and Central Americas and the Carribean
• Two morphologic forms:
o adult worms
▪ creamy white, threadlike appearance
o larvae (microfilariae)
▪ sheath - delicate transparent covering
• helpful in speciating the microfilariae forms
• adult males measure half of the typical adult female
• basic life cycle is the same for all members of the filariae
• Only one to four infective larvae - required to initiate human infection
o larvae migrate to the tissues -> development may take up to 1 year
• fertilized adult female worms -> lay live microfilariae -> reside in blood or dermis
• microfilariae exit the body via a blood meal by arthropod vector (intermediate host)
• primary method of diagnosis - microscopic examination of the microfilariae in a Giemsa-stained smear of blood or a tissue scraping of an infected nodule
• Whole blood
• Knott technique
• granulomatous lesions, eosinophilia, fever, chills, and elephantiasis, to Calabar swellings (transient swelling of subcutaneous tissues), eye involvement, and even blindness.

PARASITOLOGY RMT SOON! <3 ADM


31
CHAPTER 10: THE CESTODES
Taenia Hymenolepis diminuta Hymenolepis nana Dipylidium caninum Diphyllobothrium latum Echinococcus granulosus

Common name Taenia saginata (Beef tapeworm) Hymenolepiasis, Hymenolepiasis, Dog or cat tapeworm, fish tapeworm infection, Echinococcosis,
Taenia solium (Pork tapeworm) rat tapeworm disease dwarf tapeworm disease pumpkin seed tapeworm broad fish tapeworm hyatid cyst,
infection hyatid disease, hyatidosis
Common Taeniasis, beef tapeworm infection,
associated pork tapeworm infection
disease
Laboratory • Stool - spx of choice - recovery of • eggs in stool • eggs in stool • egg packets or gravid • eggs in stool • diagnostic stage - larval
Diagnosis eggs and gravid proglottids • Proglottids not in stool - proglottids in stool • proglottids- less stage
• Scolex – seen only after disintegrate in the frequently • Identical to Taenia
antiparasitic medication human gut • scolex – occasionally • hydatid sand
• eggs of Taenia are identical • Hydatid cyst fluid may
• gravid proglottid or scolex – for be examined on biopsy
speciation – anaphylaxis if fluid
escapes
• ELISA, indirect
hemagglutination, and
Western blot
Life Cycle Notes • ingestion of raw or undercooked • parasite of rats • Consumption of • parasite of dogs and cats • Ingestion of • Humans - accidental
beef or pork with cysticercus larva • cysticercoid larva – infective egg • humans – accidentally pleurocercoid in the intermediate hosts
• oncosphere migrates to the animal infective stage • Scolex attaches to the infected fish- precursor larval - dead-end hosts
tissue and converts into the • humans – accidental intestinal mucosa • intermediate host – stage • Sheep – typical
infective cysticercus larval stage host (zoonosis) • Adult worm is capable fleas • Adult worm is capable intermediate host
• intermediate host - cow or pig of self-reproduction • Adult worm is capable of self-reproduction • forms in the viscera
• Eggs has two routes: of self-reproduction • Coracidium – free- • dogs or wild canine -
- Passed into the swimming larva definitive host
outside by feces • requires two • Foxes are the primary
- Autoreinfection intermediate hosts: definitive hosts
• No intermediate host • Cyclops (species • Rodents - ntermediate
• cysticercoid larval stage copepod) - first hosts
may develop in intermediate host
transport hosts - fleas, • Freshwater fish –
beetles, rats which eats the
copepods become the
2nd intermediate host
• Procercoid - larval
stage - occurs in the
copepod
Epidemiology • Areas that do not practice sanitary • Worldwide • Southeastern US • worldwide • Temperate regions • Areas where sheep or
conditions and beef or pork is • Unprotected areas from • tropical and subtropical • Children - most at risk worldwide other herbivores are
eaten rats and insects are at climates • United States, Alaska raised
risk and the Great Lakes • Great Britain, parts of
• South America and South America,
Asia, Central Africa, the Australia, parts of Africa,

PARASITOLOGY RMT SOON! <3 ADM


32
Baltic region, and Asia, and China, and
Finland select portions of the
Middle East.
Clinical • Asymptomatic • Asymptomatic • Asymptomatic • Asymptomatic • Asymptomatic • Echinococcosis:
Symptoms • Taeniasis: • Hymenolepiasis: • Hymenolepiasis: • Dipylidiasis: • Diphyllobothriasis: • necrosis & buildup of
- diarrhea, abdominal pain, - diarrhea, nausea, - gastrointestinal • appetite loss, diarrhea, • digestive discomfort pressure in the tissues
change in appetite, and slight abdominal pains, and symptoms abdominal discomfort, • vitamin B12 deficiency • death may occur
weight loss anorexia. and indigestion – worm attaches to the • liver, lungs may be
- moderate eosinophilia • anal pruritus proximal part of the infected
jejunum - mimics
pernicious anemia
Treatment • Praziquantel – BEST but not used if • Praziquantel • Praziquantel • Praziquantel • praziquantel and • Surgical removal
ocular or CNS is involved • Niclosamide - • Niclosamide - • Niclosamide & niclosamide • mebendazole,
alternative alternative paromomycin - albendazole, and
alternatives. praziquantel –
alternative
Prevention and • proper sanitation practice • effective rodent control • Proper personal • dogs and cats should be • proper human fecal • personal hygiene
Control • thorough cooking of beef and pork • protection of foods hygiene examined by a disposal practices
prior to consumption • thorough inspection of • sanitation practices veterinarian • avoidance of eating • discontinue feeding
• promptly treating infected persons all potentially • Controlling transport • dogs and cats should be raw or undercooked canines with potentially
contaminated foodstuffs host populations treated and protected fish contaminated viscera
• Avoidance of contact against flea infestation • and thorough cooking • promptly treating
with potentially • children should be of all fish canines and humans
infected rodent feces taught not to let dogs or
cats lick them in or near
their mouths
Side Notes: • T. solium – cysticercosis • • • • terminal knob - • Subarctic, central
- indirect abopercular knob Europe and India
hemagglutination & • operculum - lid • Infection results in
ELISA for diagnosis structure hydatid cyst
• T. asiatica • sparganosis • Brood capsule - smaller
- eating raw pig liver, cyst within a hydatid
cattle and goat cyst
- sunken rostellum and
two rows of hooklets

NOTES:
Cestodes:
• multicellular worms, flat or ribbon-like appearance
• three morphologic forms:
o egg
▪ hexacanth embryo (oncosphere)

PARASITOLOGY RMT SOON! <3 ADM


33
• motile, first larval stage
• hooklets - six small hooks – believed to pierce the intestinal wall of the infected host
o one or more larval stages
o adult worm
• ingestion of an egg or larval stage
• intermediate host is required for the development of larva
• tegument - outer surface where the worms absorb nutrients and excrete waste
• Three distinct features common to all adult:
o Scolex
▪ defined anterior end
▪ suckers – four cup-shaped structures
▪ rostellum - fleshy extension of the scolex
o neck region
o proglottids – (strobila) individual segments with both male and female reproductive organs
• all tapeworms are self-fertilizing (hermaphroditic)
• eggs -> passed into the outside environment via the stool
• Hymenolepis nana – autoreinfection
• Echinococcus granulosus - cyst in tissue
• all the remaining intestinal tapeworm eggs consist of a hexacanth embryo with six hooklets – except D. latum

PARASITOLOGY RMT SOON! <3 ADM


34
CHAPTER 11: THE TREMATODES
Fasciolopsis buski & Fasciola hepatica Clonorchis sinensis Heterophyes heterophyes Paragonimus westermani
& Metagonimus yokogawai
Common name Large intestinal fluke, Sheep liver fluke Chinese liver fluke Heterophid fluke Oriental lung fluke
Heterophid fluke
Common associated Fasciolopsiasis, Fascioliasis, sheep liver rot Clonorchiasis Heterophyiasis Paragonimiasis, pulmonary distomiasis
disease Metagonimiasis
Laboratory Diagnosis • eggs are indistinguishable • Eggs in stool • Eggs are indistinguishable – may be • similar egg morphology with D. latum
• patient symptoms and travel history – for • Enterotest confused with C. sinensis • eggs in sputum
diagnosis • Adult worms - only seen when • Eggs in stool • commonly found in bloody samples
• recovery of the adult worm – speciation removed during a surgery or autopsy • microscopic examination – speciation • may be seen in stool samples when
• Enterotest, ELISA, and gel diffusion sputum is swallowed
Life Cycle Notes • primary difference: • ingestion of undercooked fish w/ • ingestion of contaminated • undercooked crabs or crayfish
- F. buski adults - small intestine, encysted metacercariae undercooked fish • immature flukes migrate into the
intestinal fluke • maturation in the liver • reside in the small intestine intestinal wall -> diaphragm -> lung
- F. hepatica - bile ducts, liver fluke • reside in the bile duct tissue ->
Epidemiology • ingestion of raw infected water plants • Endemic in China, Taiwan, Korea, • H. heterophyes - Africa, the Near • Asia and Africa, India, and South
• F. buski - Far East, China, Thailand, Taiwan, Vietnam, and Japan East, and the Far East. Taiwan, the America
and Vietnam, as well as regions in India • Reservoir hosts - fish-eating mammals, Philippines, Korea, Japan, Israel, and • Pigs and monkeys – reservoir hosts
and Indonesia dogs, and cats. Egypt, especially the lower Nile valley
• rabbits, pigs, and dogs - reservoir hosts • tripled to 15 million infected in China • Metagonimus - Japan, Siberia, China,
• water chestnut, lotus, and water caltrop - the Philippines, Spain, Greece, and
common food sources the Balkans
• F. hepatica - areas in which sheep and
cattle are raised
• Humans - accidental hosts
Clinical Symptoms • Fasciolopsiasis • Asymptomatic • Asymptomatic • Paragonimiasis
- abdominal discomfort • Clonorchiasis • Heterophyiasis/Metagonimiasis • pulmonary discomfort
- symptoms mimic duodenal ulcer • fever, abdominal pain, eosinophilia, • abdominal pain and discomfort, • blood-tinged sputum
- malabsorption syndrome (like giardiasis) diarrhea, anorexia, epigastric chronic mucous diarrhea and • mimic tuberculosis
- Intestinal obstruction, and even death discomfort, and occasional jaundice eosinophilia • Cerebral Paragonimiasis: Migration to
• Fascioliasis: • Enlargement and tenderness of the • eggs of both - escape into the the brain
- Biliary obstruction liver and leukocytosis lymphatics or venules via intestinal
- Eosinophilia, jaundice, liver tenderness, • Liver dysfunction wall penetration
anemia, diarrhea • migrate heart or brain
Treatment • Praziquantel • praziquantel or albendazole • praziquantel • Praziquantel
• dichlorophenol (bithionol) - F. hepatica • Bithionol - alternative
• Triclabendazole - more effective but not
available in US
Prevention and Control • exercising proper human fecal disposal • proper sanitation • avoidance of consuming undercooked • avoiding human ingestion of
• sanitation practices • fecal disposal fish undercooked crayfish and crabs
• controlling the snail population • avoiding the ingestion of raw, • proper fecal disposal • proper disposal of human waste
• avoiding the human consumption of raw undercooked, or freshly pickled products
water plants or contaminated water freshwater fish and shrimp
Side Notes: largest fluke infecting humans - F. buski

PARASITOLOGY RMT SOON! <3 ADM


35

CHAPTER 11: THE TREMATODES CONTINUATION…


Schistosoma mansoni Schistosoma japonicum Schistosoma haematobium
Common name Manson’s blood fluke Blood fluke Bladder fluke
Common associated disease • Schistosomiasis, bilharziasis, swamp fever, Katayama fever
Laboratory Diagnosis • eggs in stool or rectal biopsy • eggs in stool or rectal biopsy • eggs - concentrated urine
• ELISA
Life Cycle Notes • penetration of fork-tailed cercariae into the skin
• schistosomule migrates into the bloodstream -> maturation
• S. mansoni and S. japonicum reside in the veins that surround the intestinal tract
• S. haematobium resides in the veins surrounding the bladder
• Eggs produce enzymes that help them travel through the tissue to be excreted
• Eggs in fresh water -> miracidium -> snail -> cercariae
Epidemiology • reservoir hosts - monkeys, cattle and other livestock, rodents, dogs and catslkd
• S. mansoni – originate in the Old World; Africa; endemic areas - Puerto Rico, the West Indies, Central and South America
• S. japonicum - Far East, China, Indonesia, and the Philippines, Japan
• S. haematobium - Old World; Africa, Middle East, including Iran, Iraq, and Saudi Arabia
Clinical Symptoms • Asymptomatic
• brown hematin pigment (same with malaria px) – seen in macrophages and neutrophils of Schistosomiasis px
• abdominal pain, fever, chills, weight loss, cough, bloody diarrhea, and eosinophilia
• Painful urination and hematuria - S. haematobium
• necrosis, lesions, and granulomas
• bowel & ureter obstruction; secondary bacterial infection
• Katayama fever - systemic hypersensitivity reaction to the schistosomulae; occurs 1 to 2 months after exposure
• S. japonicum or S. haematobium - nephrotic syndrome
• S. mansoni and S. japonicum - repeated Salmonella infections
Treatment • Praziquantel
• Oxamniquine is only used for S. mansoni
• Artemisinins, artemether and artesunate – effective malarial drugs but px w/ malaria may be resistant
• Surgery
Prevention and Control • proper human waste disposal and control of the snail population
• prompt diagnosis and treatment of infected persons
• avoidance of human contact with potentially contaminated wate
• educational programs for the inhabitants of known endemic areas
• WHO recommended:
- mass treatment of everyone in a community in which there is a high prevalence and/or high risk of schistosomiasis (7 of 15 or more children
test positive)
- treatment of all children in moderately prevalent areas (2 of 15 children test positive)
- only treating diagnosed cases in low-prevalence areas
Side Notes: many species but only 5 infect humans
• Schistosoma intercalatum is found in small pockets of Africa
• Schistosoma mekongi is found in Southeast Asia
• major parasite cause of morbidity and mortality in sub-Saharan Africa
• Schistosomiasis, Bilharziasis, Swamp Fever
swimmer’s itch

PARASITOLOGY RMT SOON! <3 ADM


36

NOTES:
• class Trematoda or Digenea
• commonly known as flukes
• two groups:
o hermaphroditic
▪ infect organs, foodborne
o blood flukes or schistosomes
▪ dioecious - reproduce via separate sexes
• common life cycle:
o mollusks (snails) – intermediate host
• three morphologic forms:
o eggs
o multiple larval stages
o adult worms
• operculum – lidlike structure that flips open to release contents for further development -> Fasciolopsis and Fasciola
• spines – Schistosoma
• two categories:
o organ-dwelling
▪ intestine, bile duct, or lung
▪ all except genus Schistosoma
▪ all are hermaphroditic
▪ ingestion of metacercaria (encysted form) -> develops into adult -> self-fertilize -> eggs exit as feces or sputum
▪ eggs contact w/ fresh water -> miracidium -> snail (first intermediate host) -> larva w/ sporocysts (saclike) -> rediae -> cercariae (final-stage) -> encyst on water plants, fish, crab, or
crayfish (second intermediate host)
o blood-dwelling
▪ blood vessels around the intestine and bladder
▪ penetration of cercariae into the skin
▪ schistosomule (from cercariae) -> adult male & female -> eggs into feces or urine -> miracidium -> sporocysts -> cercariae

PARASITOLOGY RMT SOON! <3 ADM


37

CHAPTER 13: THE ARTHROPODS


Ticks Mites Spiders Scorpions Fleas
Life Cycle Notes • four morphologic stages: • Lay eggs in the burrow (skin, • less complicated than that of mites or • young scorpions are simply • genal ctenidia - of eyes, comblike
• eggs, larvae (juvenile stage), hair follicles, or sebaceous ticks smaller versions of the adult structures above the mouth parts
nymphs (developmental stage glands of hosts) and set up • female spider -> egg sack -> 200 new - develop in the ovariuterus • pronotal ctenidia – differentiation,
resembles an adult), and adults residence spiders -> adults - brood rides on the behind the head
of separate sexes • eggs –> larva –> nymph -> mother’s back until the • Flea eggs are deposited onto the
• Motile larvae – migrates in adult first molting ground -> Legless elongate larvae
blades of grass and twigs • takes approximately 2 weeks - do not change (3- to 10-day incubation period)
• Hypstome - toothed structure • Transfer requires prolonged morphology but undergo • Larvae feed on organic debris
• Anticoagulant in tick salivary close contact six moltings • Feces/excrement of adult flea has
gland • spread rapidly in - 4 to 25 years blood - additional food source
• Hard ticks – only feed once overcrowded conditions • Mature third-stage larvae (pupal) –
• Soft ticks – feed repeatedly remain up to 1 year
• Adult flea - ectoparasite
Epidemiology • Ixodes spp - primary vector for • Sarcoptes scabiei - human • venomous bite and subsequent wound • found throughout the world • Fleas are responsible for
Borrelia burgdorferi (Lyme itch or mange mite; scabies and reaction to the venom except for New Zealand and transmitting the parasites
disease) & Babesia spp. • Demodex folliculorum - hair • black widow spider – found throughout Antarctica - Dipylidium caninum,
• Dermacentor spp - Rocky follicle mite the United States • tropical and subtropical Hymenolepis nana, and
Mountain spotted fever • Trombicula akamushi - scrub • brown recluse spider - central US, climates Hymenolepis diminuta
• Amblyomma Americanum – typhus; Japan and the Far Oklahoma, Arkansas, and Missouri; • common in the desert • Xenopsylla cheopis – vector for
ehrlichiosis East reclusive, hide in clothing Southwest Yersinia pestis and Rickettsia typhi
• Ornithodoros hermsi - • Liponyssus bacoti - rat mite • hobo spider - Pacific Northwest and • Two species are of medical • may reside on dogs, cats, and
northwestern United States • Dermatophagoides spp. - western Canada concern - Hadrurus spp. rodents
and Canada allergic respiratory reactions; (desert hairy scorpion) and
• Ornithodoros turicata - house mites Centruroides spp. (bark
southwestern and midwestern scorpion)
states
Clinical • Skin reactions to the bite site • do not establish long-term • black widow spider – most venomous • venom – neurotoxin • intense itching, dermatitis,
Symptoms • Tick paralysis - (Dermacentor) residency in the host spider in North America, rarely causes a • allergic reaction to the ulcerations, and nodular swellings
• Toxemia - death • itch mites - pimple-like fatality venom, which could lead to at the bite site
lesions • brown recluse spider - necrotizing anaphylaxis and death • scratching - may result in
wound • not fatal for healthy adults secondary bacterial infection
• hobo spider • sting site will become - digit loss, tetanus or
- 50% do not inject venom painful and swollen septicemia
- Cutaneous necrosis in BRS is from • hypertension, pulmonary
HS edema, and cardiac
- Can be systemic arrhythmia
- blisters
Treatment • removal of the tick • prescription creams and • antivenin – black widow; given to • Most hospital emergency • remove the fleas aseptically with
• chloroform on the head of a lotions compromised immune systems, rooms in desert cities in the needle
tick and pulling the tick straight • clean and disinfect clothing, underlying heart conditions, or other Southwest carry scorpion • Thorough cleaning and disinfecting
out of the skin bedding, and towels health problems and to older adults antivenin of the bite site
• brown recluse - palliative care, pain • Supportive therapy, pain
management, and general wound care management, and

PARASITOLOGY RMT SOON! <3 ADM


38
• mouth parts left behind -> • hobo - simple wound care to prevent medication to control
severe tissue reactions and secondary bacterial infections anaphylaxis
secondary infection
Prevention and • protective clothing and tick • all clothing, bedding, and • minimizing places where they can hide • minimizing places where • protecting cats and dogs from fleas
Control repellents towels are washed in hot and breed they can hide and breed • lufenuron – medication; liquid in
• prophylactic vaccination water and dried in a hot • Pesticides - but not always effective • Pesticides cats, tablets for dogs
clothes dryer • Sealing holes • Sealing holes • thorough disinfection of household
• If clothes cannot be washed • Minimizing clutter outside but adjacent articles
right away, place in a plastic to the home • Public awareness
bag
Side Notes: order Ixodida • Extra-long rear pair of legs – to
family Ixodidae (hard body) move quickly
family Argasidae (soft body) •
scutum - dorsal hard shield

PARASITOLOGY RMT SOON! <3 ADM


39

CHAPTER 13: THE ARTHROPODS


Flies Lice Mosquito Bugs
Life Cycle Notes • Metamorphosis - undergo complete • lay their eggs (nits) on or very near their • lay their eggs in water • six moltings from nymph to adult
transition respective specific hosts • incubation period • Adult stages may or may not have fully
• Eggs -> larvae (multiple stages) -> adult • Head lice – hair shafts of the head and neck • four larval forms –> pupal form -> adult developed wings
• Pupa – cocoon stage • body lice - clothing fibers, chest hairs • adults are ectoparasite
• adult fly - transmits diseases • Pubic lice - pubic hair region
• 24 to 27 days – pass through 3 nymph stage
to become adult
• adult louse – lives for 30 days only
Epidemiology • found worldwide • Found worldwide • found worldwide • Cockroaches (order Blattaria) -
• Tsetse fly – Africa - Trypanosoma • transferred directly from person to person • most important group of pathogenic mechanical carriers of filth - fecal
rhodesiense, Trypanosoma gambiense • Outbreaks among schoolchildren sharing microorganism transmitters pathogens
• Sandfly - Asia, South America, Mexico, things • transmits viruses • order Hemiptera – considered true bugs
Central America, Africa, Mediterranean - • Crab lice - close personal contract, sexual • areas concentrated - fresh or brackish - bedbugs (Cimex lectularius):
Leishmania spp. intercourse water - small red marks or hemorrhagic
• Black fly - Africa, Mexico, Central and lesions.
South America - Onchocerca volvulus - triatomid bugs:
• Deer fly - Tropical Africa - Loa loa - Trypanosoma cruzi
- Chagas’ disease
- kissing bugs
Clinical • irritated and sometimes painful bite site • pediculosis - itchy papules at the • irritating dermatitis • irritating dermatitis
Symptoms • Myiasis - human tissue infestation by fly infestation site • Excessive scratching - secondary • Excessive scratching - secondary bacterial
larvae • local hypersensitivity reaction bacterial infection infection
• humans - accidental hosts • inflammation • Cockroaches do not directly cause
• maggots (larvae) – penetrates the skin • Secondary bacterial infections, mangelike symptoms
• skin lesion resembles a boil, with a small lesion • Triatomid bug bites - dermatitis
opening at the top
Treatment • Often not necessary • destroy the eggs and adults • OTC lotions and ointments - calamine • OTC lotions and ointments - calamine or
• Topical ointments • direct application of benzene hexachloride or Benadryl lotion Benadryl lotion
• Myiasis - is complete excision of the lotion
lesion with treatment to prevent
secondary bacterial infection
Prevention and • Education • proper personal • avoidance of areas of known mosquito • almost impossible to eliminate exposure
Control • wearing of protective clothing • general hygiene practices concentration • minimize the potential for exposure to
• use of screening and insect repellents • prompt treatment • use of insect repellents cockroaches and bedbugs
• Chemically treating fly breeding areas • wearing protective clothing • Eliminating garbage and increased
• using screens sanitation
• elimination of areas of standing water • vigilant surveillance for the presence of
and swampy areas bedbugs, treatment and removal
Side Notes: • abdomen is segmented - ease of • clawlike feet - grasp body hair • More than 3000 documented species • Class Insecta
movement • head louse - Pediculus humanus capitis • 10 segments – only 8 visible • Three pairs of legs and wings – found in
• body louse - Pediculus humanus humanus all bugs at some point
• crab louse - Phthirus pubis

PARASITOLOGY RMT SOON! <3 ADM


40
NOTES:
• phylum Arthropoda
• ectoparasites
• arachnids – mites, spiders, and scorpions
• insects - fleas, flies, lice, mosquitoes, and bugs
• three distinguishing characteristics:
o pairs of jointed appendages
o chitinized exoskeleton – structure on the outside of arthropod body
o hemocele - blood sac or cavity in which the blood meal is held
• 70% ethanol – preservative
• Flying insects - inactivation with chloroform or ether
• 5% formalin or sterile saline solution
• Berlese’s medium - permanent method to kill and fix specimens
• five classes of medically significant arthropods

PARASITOLOGY RMT SOON! <3 ADM


41

PARASITOLOGY RMT SOON! <3 ADM

You might also like